170
Question: 1 of 100 Time taken: 00:35 A 6-year-old boy presents with pruritic, excoriated papules on his genitalia, wrists and web spaces for the past three weeks. A scrape from a burrow in his web spaces reveals this picture. What is your diagnosis? (Please select 1 option) A. Ctenocephalides canis (dog flea) bites Incorrect answer selected B. Fire ant bites C. Molluscum contagiosum D. Pediculosis E. Scabies This is the correct answer

Dermatology 100+11

Embed Size (px)

DESCRIPTION

dermatology

Citation preview

Question: 1 of 100

Time taken: 00:35

A 6-year-old boy presents with pruritic, excoriated papules on his genitalia, wrists and web spaces for the past three weeks.

A scrape from a burrow in his web spaces reveals this picture.

What is your diagnosis?

(Please select 1 option) A. Ctenocephalides canis (dog flea) bites Incorrect answer selected

B. Fire ant bites

C. Molluscum contagiosum

D. Pediculosis

E. Scabies This is the correct answer

The picture shows a typical sarcoptes scabies mite as seen under the microscope.

Common in children, patients present with extremely pruritic papules, especially on the genitalia, web spaces and wrists.

Options A, B, C and D: These other conditions do not present with these symptoms and are not represented by the picture.

Question: 2 of 100

Time taken: 02:19

A 10-year-old boy presents with this lesion (shown) for the past three months. It is asymptomatic.

What is the etiological cause?

(Please select 1 option) A. Bartonella henselae Incorrect answer selected

B. Molluscum contagiosum virus

C. Herpes simplex virus

D. Human papilloma virus This is the correct answer

E. Trichophyton verrucosum

The patient has a viral wart, caused by the human papilloma virus.

Option A: Bartonella henselae causes bacillary angiomatosis.

Option B: Molluscum contagiosum virus is poxvirus that causes another common cutaneous infection, molluscum contagiosum.

Option C: Herpes simplex virus causes herpetic infections, for example, herpes labialis, herpes genitalis, herpetic whitlow.

Option E: Trichophyton verrucosum is a dermatophyte which causes tinea.

Question: 3 of 100

Time taken: 05:20

This 7-year-old boy has this recurrent rash on his flexures.

Which organism is the commonest cause of exacerbation of this condition?

(Please select 1 option) A. Staphylococcus aureus Correct

B. Varicella-zoster virus

C. Human papilloma virus

D. Pseudomonas aeruginosa

E. Trichophyton mentagrophytes

The patient has atopic dermatitis affecting mainly his flexures.

Patients with atopic dermatitis commonly experience exacerbations of their condition due to infection with Staphylococcus aureus. Treatment would then include a course of anti-Staph antibiotics.

Options B-E: These infections are not particularly associated with worsening of atopic dermatitis.

Question: 4 of 100

Time taken: 05:51

On skin examination of a patient you notice multiple, small, raised, slightly dome-shaped lesions containing purulent material.

Which of the following is the best description of these lesions?

(Please select 1 option) A. Bullae Incorrect answer selected

B. Fissures

C. Pustules This is the correct answer

D. Telangiectasia

E. Ulcers

Pustules are small elevations of the skin containing cloudy or purulent material, usually consisting of necrotic inflammatory cells.

Option A: Bullae are large vesicles containing serous fluid.

Option B: Fissures are cracks in the skin that are narrow but deep.

Option D: Telangiectasias are collections of enlarged capillaries visible on the skin or mucous membranes.

Option E: Ulcers are seen as discontinuity of the skin with complete loss of the epidermis and often portions of the dermis and subcutaneous fat.

Question: 5 of 100

Time taken: 05:59

A 15-year-old boy presents with a three day history of a generalised painful rash associated with erosions in his mouth and genitals.

He was recently started on carbamazepine for seizures. He is febrile and appears ill.

On examination, there are dusky, targetoid papules and plaques on his face, trunk and limbs.

Firm pressure on the dusky areas leads to desquamation of the superficial skin.

What sign is being demonstrated?

(Please select 1 option) A. Auspitz's sign Incorrect answer selected

B. Darier's sign

C. Dermatographism

D. Koebner phenomenon

E. Nikolsky's sign This is the correct answer

This patient has toxic epidermal necrolysis (TEN), a severe idiosyncratic drug reaction, with a high morbidity and mortality rate.

Commonly implicated drugs include anti-epileptics, penicillins, NSAIDS and allopurinol.

Patients present with erythematous-to-dusky plaques and targetoid lesions that progress rapidly to painful, full-thickness, epidermal desquamation involving more than 30% of body surface area. Mucosal involvement is also seen.

Nikolsky's sign is elicited when rubbing of the skin results in exfoliation of the outermost layer and is classically seen in patients with TEN.

Option A: Auspitz's sign is seen when psoriatic scales are scraped off leaving punctuate bleeding points.

Option B: Darier's sign is seen in patients with mastocytosis or urticaria pigmentosa, whereby rubbing of the lesions leads to urticarial-like wheals.

Option C: Dermatographism is a form of urticaria whereby rubbing of the skin leads to development of wheals.

Option D: Koebner's phenomenon is the appearance of skin lesions in areas subjected to trauma. It is classically seen in psoriasis but may also be seen in other dermatoses, for example, lichen planus, vitiligo, Darier's disease.

Question: 6 of 100

Time taken: 06:14

A 10-year-old boy presents with these lesions since birth. His father and older brother have similar lesions.

What is the inheritance pattern of this condition?

(Please select 1 option) A. Autosomal dominant Correct

B. Autosomal recessive

C. Sporadic

D. X linked dominant

E. X linked recessive

The patient has neurofibromatosis type 1 (NF1), caused by a mutation in the NF1 gene. The condition is inherited as an autosomal dominant condition. However, many cases present as a new mutation.

Criteria for diagnosis of NF1:

Six or more café au lait spots or hyperpigmented macules greater than or equal to 5 mm in diameter in children younger than 10 years and to 15 mm in adults

Axillary or inguinal freckles

Two or more typical neurofibromas or one plexiform neurofibroma

Optic nerve glioma

Two or more iris hamartomas (Lisch nodules), often identified only through slit-lamp examination by an ophthalmologist

Sphenoid dysplasia or typical long bone abnormalities such as pseudarthrosis

First-degree relative (for example, mother, father, sister, brother) with NF1.

Options B, C, D and E: Neurofibromatosis type 1 does not follow these patterns of inheritance.

Question: 7 of 100

Time taken: 06:38

A 6-year-old boy presents with these lesions on one side of his trunk for the past two months. They appear to be spreading but are otherwise asymptomatic.

What is your diagnosis?

(Please select 1 option) A. Keratosis pilaris Incorrect answer selected

B. Acne vulgaris

C. Molluscum contagiosum This is the correct answer

D. Viral warts

E. Tinea

Molluscum contagiosum is a common cutaneous viral infection seen in children. It is caused by a poxvirus.

It presents as small skin-coloured dome-shaped papules with central umbilication. They usually occur unilaterally. They may take one to two years to resolve, even without treatment.

Options A, B, D and E: These options do not present with these dome-shaped, skin coloured papules.

Question: 8 of 100

Time taken: 07:02

This 7-year-old boy has this recurrent rash on his flexures.

Which organism is the commonest cause of exacerbation of this condition?

(Please select 1 option) A. Staphylococcus aureus Correct

B. Varicella-zoster virus

C. Human papilloma virus

D. Pseudomonas aeruginosa

E. Trichophyton mentagrophytes

The patient has atopic dermatitis affecting mainly his flexures.

Patients with atopic dermatitis commonly experience exacerbations of their condition due to infection with Staphylococcus aureus. Treatment would then include a course of anti-Staph antibiotics.

Options B to E: These infections are not particularly associated with worsening of atopic dermatitis.

Question: 9 of 100

Time taken: 07:23

Which of the following is/are recognised causes of alopecia in children?

True / False Alopecia areata. Correct

Ectodermal dysplasia. Correct

Naevus flammeus. Incorrect answer selected

Ringworm. Correct

Trichotillomania. Correct

Alopecia areata is autoimmune destruction of hair follicles.

Ectodermal dysplasia is characterised by absent or deficient function of at least two derivatives of the ectoderm, for example, teeth and hair.

Trichotillomania is an impulsive need to pull out hair from scalp and eyelashes.

Ringworm is a fungal infection which results in hair loss.

Naevus flammeus is another name given to a port-wine stain.

Question: 10 of 100

Time taken: 07:38

Which of the following therapies are appropriate in children for the condition indicated?

True / False 1% hydrocortisone for infantile eczema Correct

25% benzyl benzoate for scabies Incorrect answer selected

Coal tar for psoriasis Correct

Ketoconazole for alopecia areata Incorrect answer selected

Surgical excision for a cavernous haemangioma 3 cm and 4 cm on the arm Incorrect answer selected

Permethrin is preferred for scabies, with malathion as second line. Benzyl benzoate is an irritant.

Cavernous haemangiomas are usually not present at birth but appear in the first two weeks of life.

Lesions are usually on the face, neck or trunk and are well-circumscribed and lobulated. Treatment options do not include surgical excision. Treatment may be indicated if there is inhibition of normal development - for example impairing normal binocular visual development by obstructing the vision from one eye. It may involve systemic or local steroids, sclerosants, interferon, or laser treatment.

Alopecia areata is an autoimmune condition causing discrete areas of hair loss. Treatment options include cortisone injections into the affected areas, and the use of topical cortisone creams.

Question: 11 of 100

Time taken: 11:55

A young child is seen with peeling of the palms of the hands and the soles of the feet.

Which of the following is a potential explanation for this presentation?

(Please select 1 option) Chickenpox Incorrect answer selected

Kawasaki disease. This is the correct answer

Measles

Mumps

Urticaria

The secondary viraemia of varicella, featuring viral particles being spread to the skin 14-16 days after initial exposure, causes the typical vesicular rash. Skin peeling is not a feature.

The histopathologic pattern in papular urticaria consists of

Mild subepidermal oedema

Extravasation of erythrocytes

Interstitial eosinophils, and

Exocytosis of lymphocytes.

The reaction is thought to be caused by a haematogenously disseminated antigen deposited by an arthropod bite in a patient who is sensitive.

In Kawasaki's disease, desquamation of the fingers and toes begins in the periungual region, may involve the palms and soles, and usually is observed one to two weeks after the onset of fever.

Tinea infection is associated with peeling of the skin in the affected area, and in scarlet fever, the skin rash is associated with streptococcal Group A infection; the skin begins to peel usually around the sixth day of the rash.

Atopic eczema is frequently associated with ichthyosis and desquamation.

Kawasaki disease is associated with desquamation and mucous membrane involvement.

Question: 12 of 100

Time taken: 12:03

A 6-month-old boy is brought to clinic by his parents. He had a fever for three days, and as this disappeared he was noted to have a rash.

On examination he is noted to be apyrexial, but has a macular rash on the trunk and lower limbs.

Which of the following is the most likely diagnosis?

(Please select 1 option) Erythema multiforme Incorrect answer selected

Henoch-Schönlein purpura

Idiopathic thrombocytopenia

Meningococcal septicaemia

Roseola infantum This is the correct answer

Roseola infantum is due to herpes virus 6 and characteristically causes a macular rash with lymphadenopathy.

Erythema multiforme characteristically causes target lesions with blistering.

Idiopathic thrombocytopenia causes a petechial rash.

Henoch-Schönlein purpura causes a purpuric rash on the buttocks and lower limbs.

Meningococcal septicaemia causes a non-blanching purpuric rash.

Question: 13 of 100

Time taken: 12:33

Theme:Paediatric DermatologyA Alopecia areata

B Dermoid cyst

C Eczema

D Erythem multiforme

E Erythema nodosum

F Henoch-Schonlein purpura

G Kawasaki disease

H Keloid

I Monilia

J Napkin rash

K Pigmented naevi

L Seborrhoeic dermatitis

M Urticaria

For each clinical scenario select the most appropriate diagnosis:

An 11-year-old girl is referred to the school nurse with an intensely itchy, erythematous rash in her flanks. She has scratched them and the nurse notices residual bruising and flaring.

Incorrect - The correct answer is Urticaria

This girl has urticaria, a common allergic manifestation, but the offending allergen is often not identified. Some recurrent cases are caused by sensitivity to food colouring agents.

The parents of a 3-year-old girl present to their GP worried that their daughter has hair loss. On examination the GP notes localised hair loss on the scalp with a characteristic margin of exclamation mark hair shafts.

Incorrect - The correct answer is Alopecia areata

This girl has alopecia areata. It is usually self-limiting, but occasionally may progress to alopecia totalis.

The parents of a 4-year-old boy of Jamaican origin present to their GP after noticing that their son has developed an excessively large scar on his knee when he fell-over whilst running six-weeks previously.

Incorrect - The correct answer is Keloid

This boy has keloid scarring. Treatment is difficult and surgical excision is rarely attempted because of the tendency to increase keloid formation.

The parents of an 18-month-old girl present to their GP worried that their daughter has numerous small spots in the region of her nappy. On examination the GP finds multiple erythematous, ulcerated, satellite lesions in her groin.

Incorrect - The correct answer is Monilia

This baby has monilia, which is caused by Candida.

The parents of a 5-year-old Japanese boy present to the GP worried after their son developed a fleeting erythematous rash associated with a fever, which has been present for five days. On examination the GP notes that the boy has cracked lips, a red tongue, swollen hands and swollen glands in his neck.

Incorrect - The correct answer is Kawasaki disease

This boy has Kawasaki disease, also called mucocutaneous lymph node syndrome. To reduce the risk of the development of coronary artery aneurysms treatment includes intravenous gamma globulin and aspirin.

Question: 14 of 100

Time taken: 12:45

Regarding atopic eczema which of the following statements is/are true?

True / False Does not have a genetic basis. Incorrect answer selected

Is a generalised rash over the whole body. Incorrect answer selected

Never benefits from dietary measures. Incorrect answer selected

Should be treated in its early stages with topical corticosteroids. Incorrect answer selected

Usually starts in the first year of life. Correct

The typical distribution is face, ears, elbows and knees.

Cow's milk is a common cause and switching to soya milk for instance may assist.

It is more common in those with a family history of asthma, hayfever and eczema.

Topical steroids should be applied sparingly only if symptoms cannot be controlled.

What is atopic eczema?...

Question: 15 of 100

Time taken: 13:03

Which of the following statements is/are true regarding children with eczema?

True / False Are more often found in social class V families. Incorrect answer selected

May develop adrenal suppression from topical steroid application. Correct

Often have a family history of psoriasis. Incorrect answer selected

Often have involvement of the face. Correct

Should not use "bubble baths". Correct

Eczema is related to the presence of a family history of atopic condtions rather than social class or psoriasis occurring in family members.

The common sites of occurrence are in the face, ears, elbows and knees.

Irritant chemicals may exacerbate the rash.

Topical steroids should be used only if symptoms do not resolve as they may lead to adrenal suppression.

Question: 16 of 100

Time taken: 13:12

A 2-year-old child develops a scaly macular diffuse rash and is diagnosed with psoriasis.

Which of the following is true of this diagnosis?

(Please select 1 option) A common cause of alopecia totalis Incorrect answer selected

Associated with cows' milk protein intolerance

Best treated initially with steroid creams

Sometimes a sequel to staphylococcal infection

Typically not itchy This is the correct answer

Eczema is associated with cows' milk intolerance and itchiness, rather than psoriasis. Coal tar is commonly used initially.

Small patches of alopecia are associated with psoriasis, rather than complete alopecia. Telogen effluvium is the typical form of hair loss that psoriasis induces, but sometimes psoriasis can cause a scarring alopecia.

There is a rather loose association between streptococcal throat infections and psoriasis.

Further reading:

Telogen Effluvium eMedicine

Question: 17 of 100

Time taken: 13:20

A 17-year-old boy is diagnosed with scabies.

Which of the following statements regarding scabies is correct?

(Please select 1 option) Is best treated by salicylate emulsion Incorrect answer selected

It can be spread by a droplet infection

It causes itchiness in the skin even where there is no obvious lesion to be seen This is the correct answer

It is caused by Staphylococcus aureus

Typically affects the face

Scabies is an infestation of the skin with the microscopic mite Sarcoptes scabei. Infestation is common, found worldwide, and affects people of all races and social classes.

Scabies spreads rapidly under crowded conditions where there is frequent skin-to-skin contact between people, such as in hospitals, institutions, child-care facilities, and nursing homes.

Scabies can spread by direct, prolonged, skin-to-skin contact, with a person already infested with scabies. Contact must be prolonged (a quick handshake or hug will usually not spread infestation).

Infestation is easily spread to sexual partners and household members. Infestation may also occur by sharing clothing, towels, and bedding.

Scabies is characterised by papular-like irritations, burrows or rash of the skin, especially the webbing between the fingers; the skin folds on the wrist, elbow, or knee; the penis, the breast, or shoulder blades.

A number of treatments are available for the treatment of scabies, including permethrin ointment, benzyl benzoate, and oral ivermectin for resistant cases.

Antihistamines and calamine lotion may be used to alleviate itching.

Question: 18 of 100

Time taken: 13:29

A young child is brought to clinic with severe eczema.

Which of the following statements concerning his treatment is correct?

(Please select 1 option) Might benefit from a diet free of cows' milk Correct

Should be given a course of oral steroids

Should be treated with the aim of complete cure

Should not be immunised against measles

Should not be immunised against pertussis

Cows' milk allergy may precipitate severe eczema, and trial of soy based formula may have beneficial effects on the infant's condition.

Complete cure is not always a practical aim of treatment.

Amelioration of symptoms, using appropriate preventative measures and topical preparations, may minimise, but not totally eradicate the condition.

Most infants grow out of the condition by the time they are 2-3 years old.

There is no current evidence to suggest that infants with eczema should not receive measles or pertussis immunisation, but they should not be immunised if there is a concurrent skin infection.

Oral steroids are a last resort of treatment and are only rarely used in infants with severe eczema.

Question: 19 of 100

Time taken: 13:36

A 4-month-old child is diagnosed with napkin rash.

Which of the following is the most appropriate treatment of mild napkin rash?

(Please select 1 option) Bactroban cream Incorrect answer selected

Exposure to air This is the correct answer

Oral flucloxacillin

Topical betamethasone

Topical iodine

Causes of napkin rash include contact dermatitis, which may produce ammonia and this may burn the skin.

Infection with bacteria and candida yeasts may cause nappy rash, as can psoriasis and atopic dermatitis affecting the nappy area.

Nappy rash is not an indicator of infantile eczema, and it is not more common in boys, nor is it less common in soy fed infants.

Treatment is best achieved by prevention with frequent (disposable absorbent) nappy changing, and fluid feeding early in the day to lessen night time urination.

Anti-fungal lotions may also be useful.

Question: 20 of 100

Time taken: 13:43

A 3-year-old child is noted to have a lesion on the left cheek and a diagnosis of impetigo is considered.

Which one of the following statements is true regarding impetigo?

(Please select 1 option) Characterised by scab-covered weeping lesions Correct

Highly infectious requiring isolation

Restricted to the face

Usually caused by Candida albicans

With dry, itchy and scaly lesions

Impetigo is a skin infection, caused by Staphylococcus aureus, Streptococcus pyogenes, or both.

It leads to the formation of scabby, yellow-crusted sores and, sometimes, small blisters filled with yellow fluid.

It can occur anywhere in the body, but commonly occurs in the face, arms or legs.

It is a highly contagious, but does not require isolation.

Question: 21 of 100

Time taken: 13:56

Are the following recognised findings in psoriasis?

True / False Angular stomatitis Incorrect answer selected

Dystrophy of nails Correct

Iridocyclitis Incorrect answer selected

Loss of hair Incorrect answer selected

Response to chloroquine Incorrect answer selected

Psoriatic changes include

Dystrophy of the nails

Arthropathy which is often oligoarthropathy, and

Typical skin changes.

Eye changes are not a feature however.

Unlike systemic lupus erythematosus (SLE), hair loss/alopecia is not a typical feature of psoriasis nor is angular stomatitis expected.

Chloroquine is contra-indicated in psoriasis due to a recognised association with erythroderma.

Information on psoriasis

Question: 22 of 100

Time taken: 14:16

Theme:HairA Aarskog syndrome

B Branchio-oculo-facial syndrome

C Cornelia De Lange syndrome

D EEC syndrome

E Fetal hydantoin syndrome

F Klippel-Feil syndrome

G Menkes syndrome

H Retinoic acid embryopathy

I Rubenstein-Taybi syndrome

J Waardenburg syndrome

The listed conditions are associated with abnormalities of hair.

For the below descriptions, choose the condition which best fits.

A 2-month-old girl is investigated for dysmorphic features. She has bushy eyebrows, synophrys and long curly lashes.

Incorrect - The correct answer is Cornelia De Lange syndrome

A 4-month-old baby is admitted with status epilepticus. He is noted to have sparse hair, which when magnified appears fractured and twisted.

Incorrect - The correct answer is Menkes syndrome

A 9-year-old boy is investigated for short stature. He is noted to have hypertelorism, anteverted nostrils and his hair demonstrates a 'widows peak'. He also has genital abnormalities.

Incorrect - The correct answer is Aarskog syndrome

Cornelia de Lange syndrome is a condition of unknown aetiology.

Clinically patients have a marked retardation of growth, evident at birth and as a result patients fail to thrive. Patients also have developmental delay and learning difficulties.

Facial features include microcephaly, bushy eyebrows and synophrys and a thin down turning upper lip.

Patients also exhibit abnormalities of limbs, in particular micromelia, phocomelia oligodactyly and syndactyly.

Menkes syndrome is thought to be X- linked recessive.

It is a condition of progressive cerebral deterioration, beginning in early infancy resulting in death by the age of three.

The disease results from an abnormality of copper transport and low levels of copper and caeruloplasmin are found in all patients.

Manifestations include sparse lightly pigmented hair which shows twisting (pili torti) and breakages.

Neurological deficits are profound with irritability, hypertonia and seizures as well as severe neurological degeneration.

Aarskog syndrome is also a condition inherited as a result of X-linked recessive inheritance.

It is characterised by brachydactyly , hypertelorism and 'shawl scrotum'.

Growth deficiency may be of prenatal onset, resulting in failure to thrive.

Question: 23 of 100

Time taken: 14:35

Theme:SkinA Angelman syndrome

B Chondrodysplasia punctata

C EEC syndrome

D Homocystinuria

E Hypomelanosis of Ito

F Incontinentia pigmenti syndrome

G Maffucci syndrome

H Peutz-Jegher syndrome

I Waardenburg syndrome

J Xeroderma pigmentosa syndrome

The listed conditions are associated with altered skin pigmentation.

For the below descriptions choose the condition which best fits:

A neonate develops seizures. She is noted to have blisters in a linear distribution. The blisters disappear to leave hyper-pigmentation in the form of whorls and streaks.

Incorrect - The correct answer is Incontinentia pigmenti syndrome

A 7-year-old boy has a severe hearing impairment. He is fair skinned, with white forelock and his eyes show heterochromia.

Incorrect - The correct answer is Waardenburg syndrome

A 14-year-old boy is referred to growth clinic. He is tall and slim with a kyphoscolosis and arachnodactyly. He has a molar flush with patchy red blotches elsewhere.

Incorrect - The correct answer is Homocystinuria

Incontinentia pigmenti is an X linked dominant condition.

Babies present in infancy with bullous skin lesions which progress to vesicular lesions before becoming pigmented. The pigmentation then develops a marbled streaky pattern and this is most apparent on the trunk.

Other features include dental anomalies and patchy alopoecia. Patients have an increased risk of malignancy.

Waardenburg syndrome is an autosomal dominant condition. Features include partial albinism (usually evident as a white forelock) and deafness which is the more serious feature.

The defect appears to be in the organ of Corti.

The hearing impairment may be unilateral or bilateral.

Other features include heterochromia, pigmentation on retinae and vitiligo.

Question: 24 of 100

Time taken: 14:47

Can the following conditions produce a pruritic rash?

True / False Dermatitis herpetiformis Correct

Eczema Correct

Erythema nodosum Incorrect answer selected

Granuloma annulare Incorrect answer selected

Lichen planus Correct

Granuloma annulare is a benign, non-pruritic condition that presents with erythematous, firm nodules which coalesce to form ring-shaped lesions.

Lichen planus produces an intensely pruritic rash that shows the Koebner phenomenon in that scratching produces erythematous, pruritic papules and vesicles.

Dermatitis herpetiformis produces a vesicular pruritic rash and is associated with coeliac disease.

Erythema nodosum produces a red painful nodular rash often on the fronts of the shins.

Question: 25 of 100

Time taken: 15:02

Theme:Fungal Skin InfectionsA Candida

B Favus

C Kerion

D Onychomycosis

E Pityriasis versicolor

F Tinea capitis

G Tinea cruris

H Tinea manuum

I Tinea pedis

J Tinea incognito

For each patient below, choose the single most likely diagnosis from the list of options.

Each option may be used once, more than once or not at all.

An 8-year-old boy presents with a coin lesion on the scalp. The hairs are broken off and the lesion is scaly.

Incorrect - The correct answer is Tinea capitis

Tinea capitis or scalp ringworm is common in school children and affects boys more than girls. Sometimes epidemics occur in schools. The commonest types due to human fungi present as multiple coin lesions with stubs of broken hair and a clear centre.

A 10-year-old boy lives with his father on an animal farm. He presents with a large boggy swelling on the scalp associated with hair loss. The post auricular and occipital nodes are enlarged.

Incorrect - The correct answer is Kerion

Kerion is an uncommon type of tinea capitis and is caused by animal fungi. There is severe inflammation resulting in a large boggy pus filled swelling. It may result in permanent hair loss.

Question: 26 of 100

Time taken: 15:18

Theme:Common skin infectionsA Cellulitis

B Herpes simplex

C Herpes zoster

D Intertrigo

E Molluscum contagiosum

F Oral candidiasis

G Tinea capitis

H Tinea pedis

I Scabies

J Viral warts

For each patient below, choose the single most likely diagnosis from the list of options.

Each option may be used once, more than once or not at all.

A GP is notified of a school in which a number of children have multiple coin shaped lesions on their scalps associated with itching, hair loss and scaling.

Incorrect - The correct answer is Tinea capitis

Tinea capitis is an infectious disease occuring chiefly among school children. Boys are affected more than girls, however in epidemics caused by trichophyton tonsurans the sex incidence is equal. Trichophyton tonsurans is now the commonest cause of scalp ringworm in the UK and USA.

An 8-year-old girl presents to the out-patients department with multiple verrucous lesions on the sides of her fingers and face. She had a renal transplant three years ago.

Incorrect - The correct answer is Viral warts

Viral warts are common in children and spread by innoculation into areas where the skin barrier is broken. They spontaneously resolve in immunocompetent individuals and tend to be extensive and recalcitrant in the immunocompromised.

Question: 27 of 100

Time taken: 15:28

A 3-year-old boy presents with an itchy rash.

This has been present on his wrists and hands for three weeks and disturbs his sleep. He was born at 37+2/40 weighing 3.56 kg and there were no neonatal problems. He is fully immunised and on no medication. His 5-year-old sister has a similar rash.

On examination his temperature is 36.4°C, RR 20/min and HR 90/min. He has red raised papules around his wrist and small linear skin breaks in the finger webs. Otherwise there are no abnormalities to find.

What is the most likely diagnosis?

(Please select 1 option) Ammoniacal dermatitis Incorrect answer selected

Atopic dermatitis

Impetigo

Napkin candidiasis

Scabies This is the correct answer

The history of itchy papular rash worst at night, affected sibling, and finding of burrows all point to scabies as the diagnosis.

As the female mites burrow and bury their eggs and faeces, toxic and antigenic substances cause the condition.

Question: 28 of 100

Time taken: 15:44

A 2-month-old girl presents with a nappy rash. This has gradually worsened over the past 1-2 weeks. Otherwise she has been well.

She was born at term weighing 3.01 kg and there were no neonatal problems. She has received her first set of routine immunisations and is on no medications. There is no family history of note.

On examination her temperature is 36.8°C, respiratory rate 35/min and heart rate 110/min. She is completely well apart from a symmetrical confluent red rash in the napkin area. This has a peeling border, with 2-3 mm circular red spots with peeling borders lateral to it.

What is the most likely diagnosis?

(Please select 1 option) Ammoniacal dermatitis Incorrect answer selected

Histiocytosis

Napkin candidiasis This is the correct answer

Scabies

Seborrhoeic dermatitis

The rash suggests candida infection in the nappy area.

This may be isolated or complicate an ammoniacal nappy rash.

The latter is rarely seen in these days of disposable nappies, but spares the flexures.

The satelite lesions and peeling edge are very characteristic of candida.

The rash responds to topical anticandidals.

Question: 29 of 100

Time taken: 15:54

A 10-month-old girl is referred with a florid skin rash.

Soon after birth she developed cradle cap and dry skin on the cheeks. This gradually improved, but over the past few months she has had dry red skin over the trunk, the elbows, wrists, knees, ankles and base of the ears. This is itchy and disturbs sleep.

She was born at 37+3/40 weighing 3.34 kg and there were no other neonatal problems. She has been fully immunised. Mother has hay fever and asthma.

On examination she is on the 10th centile for height, weight and OFC. Temperature is 36.4°C, RR 30/min and HR 100/min. She has a dry red scaly rash on patches of the trunk, elbows, wrists, ankles and knees. She has enlarged glands in the axillae and groins.

What is the most likely diagnosis?

(Please select 1 option) Ammoniacal dermatitis Incorrect answer selected

Atopic dermatitis This is the correct answer

Contact dermatitis

Psoriasis

Seborrhoeic dermatitis

The type, distribution and progress of the rash, in conjunction with the family history, suggest atopic eczema.

This is treated with moisturisation and topical anti-inflammatories (steroids), the strength of the latter being titrated against effect.

Irritants and dessicants should be avoided.

Question: 30 of 100

Time taken: 16:02

A 12-year-old boy presents with a florid rash on the hands and feet.

He became ill three days before, when he developed painful mouth ulcers. Yesterday he developed a rash on the hands and feet. He was a full term normal delivery, and previously has been very healthy. He is on no medications, is fully immunised, and there is no family history of note.

On examination he has a temperature of 38.5°C, respiratory rate 15/min and heart rate is 85/min. He has profuse oral ulcers on his tongue and buccal mucous membranes. He has many circular raised pinky red lesions over the hands and feet with darker centres. He has tender cervical lymphadenopathy.

What is the most likely diagnosis?

(Please select 1 option) Erythema multiforme Correct

Hand foot and mouth disease

Infectious mononucleosis

Kawasaki disease

Staphylococcal scalded skin syndrome

The history suggests an attack of oral ulceration, followed by the development of target lesions over the extremities.

The likely diagnosis is primary herpes 1 infection, complicated by erythema multiforme.

Treatment is supportive, though acyclovir may be given to control the herpes.

Question: 31 of 100

Time taken: 16:09

A 4-year-old boy presents with a perioral rash. This has been present for many months, but is not resolving.

He was born at term and there were no neonatal problems. He has been well and thriving on no medications. He is fully immunised to date and there is no FH/SH of note.

On examination his temperature is 36.97°C, heart rate 95/min and respiratory rate of 20/min. He has dry flaky skin around his mouth, which he licks frequently.

What is the most likely diagnosis?

(Please select 1 option) Contact dermatitis Correct

Erythema multiforme

Impetigo

Kawasaki disease

Primary herpes simplex

The history of local dryness associated with licking is a form of contact dermatitis.

Recurrent contact with irritants such as saliva, bubble bath, detergents, soaps and citrus juices inflame the skin.

Sweat may result in a similar foot problem.

Treatment is with emollients and avoidance of the trigger.

Question: 32 of 100

Time taken: 16:17

A 7-month-old girl presents with fever and a rash.

She was completely well till five days ago, when she developed a slight cold. The next day she developed fever to 39.7°C, which has persisted despite antipyretics. Despite this she has remained relatively well and continues to drink, though her appetite is poor. Today she has developed a rash over the face and trunk.

She was born at term weighing 3.8 kg and there were no neonatal problems. She is fully immunised to date and there is no family history or social history of note.

On examination she has a temperature of 36.8°C, respiratory rate 25/min and heart rate 100/min. The rash is macular, profuse, pink and blanching. It is most prominent over the face and trunk. She has shotty cervical lymphadenopathy.

What is the most likely diagnosis?

(Please select 1 option) CMV infection Incorrect answer selected

Infectious mononucleosis

Measles

Parvovirus infection

Roseola infantum This is the correct answer

The history of a well child with high fever for a few days followed by resolution of fever at around the time of appearance of a rose-coloured rash is characteristic of roseola infantum.

Since the introduction of measles, mumps, rubella vaccine (MMR), this is by far the commonest cause of a measles-like rash.

The peak incidence is 6-18 months. Five percent develop febrile seizures.

It is caused by human herpesvirus 6 and 7.

Question: 33 of 100

Time taken: 16:25

A 17-year-old pregnant female attends antenatal clinic and is noted to have scattered small, raised lesions on her trunk and axillary freckles.

She was not aware of any of her family members having these lesions.

What is the likely mode of inheritance of this condition?

(Please select 1 option) Autosomal dominant Correct

Autosomal recessive

Trinucleotide repeating

X-linked dominant

X-linked recessive

This patient has neurofibromatosis with axillary freckling and neurofibromas.

This is usually inherited as autosomal dominant although it may arise from a sporadic new mutation of the NF1 gene.

Question: 34 of 100

Time taken: 16:43

Theme:Mucocutaneous infections and infestationsA Enterobius vermicularis

B Chlamydia

C Candidiasis

D Tinea pedis

E Pityriasis rosea

F Scabies

G Ringworm

H Lichen sclerosis

I Napkin dermatitis

J Lichen planus

For each of the following children who present with an itch, select the most appropriate diagnosis from the list of options?

A child has itching in the vulva region mostly at night.

Correct

Enterobius vermicularis is also known as threadworms. It is a common infestation in children and presents with nocturnal anal pruritus and a perianal irritation.

A child presents with annular regions over the trunk.

Incorrect - The correct answer is Ringworm

Ringworm also known as tinea corporis presents with plaques of scaling eczema which are characteristically itchy.

A diabetic child presents with a vulva rash.

Correct

Candidiasis is caused by Candida albicans (yeast). In babies it presents as a perineal rash which usually affects the flexures. Satellite lesions may also be seen.

Question: 35 of 100

Time taken: 16:54

A 10-month-old child presents acutely with non-blanching, purple skin lesions.

In these circumstances, which of the following statements is true?

(Please select 1 option) A haemoglobin of 7.3 g/dl, with a white cell count of 12 x 109/l and platelets of 86 suggests acute leukaemia Correct

If the haemoglobin is 12.3 g/dl, the white cell count is 12 x 109/l, and the platelets are 97, this suggests Henoch-Schönlein purpura

Microscopic haematuria suggests infective endocarditis

Splenomegaly suggests idiopathic thrombocytopaenic purpura

The likeliest cause is meningococcal disease

Causes of purpura in children include:

Thrombocytopaenic:

Impaired production: leukaemia, aplastic anaemia

Excessive destruction: immune (ITP), secondary (systemic lupus erythematosus [SLE], drugs, viral infections), alloimmune neonatal thrombocytopaenia

Consumptive coagulopathy: disseminated intravascular coagulation (DIC), haemolytic uraemic syndrome, thrombotic thrombocytopaenic purpura

Congenital: giant haemangioma, Wiskott-Aldrich syndrome.

Vascular disorders:

Congenital: connective tissues disorders (osteogenesis imperfecta, Ehlers-Danlos, Marfan's)

Acquired: meningococcal and other severe infections

Immune: Henoch-Schönlein purpura (HSP), connective tissue disorders (SLE)

Drugs.

Although meningococcal disease is an important differential in all children with purpura, it is not the most likely cause. Even in febrile children, only 7% will have meningococcal disease.

The platelet count in HSP is normal.

In idiopathic thrombocytopaenic purpura there is an absence of hepatosplenomegaly.

Acute leukaemia is an important differential in anybody with pancytopaenia; 2/3 cell lines affected also make this most likely.

Microscopic haematuria plus purpura may occur in infective endocarditis, but may also be related to HSP or SLE.

Question: 36 of 100

Time taken: 17:03

A 10-month-old child presents acutely with non-blanching, purple skin lesions.

In such circumstances which of the following statements is/are true?

True / False A haemoglobin of 7.3 g/dl, with a white cell count of 12 x 109/L and platelets of 86 suggests possible acute leukaemia. Correct

If the haemoglobin is 12.3 g/dl, the white cell count is 12 x 109/L, and the platelets are 97, this suggests Henoch-Schonlein purpura. Incorrect answer selected

Microscopic haematuria suggests infective endocarditis. Incorrect answer selected

Splenomegaly suggests idiopathic thrombocytopenic purpura. Incorrect answer selected

The likeliest cause is meningococcal disease. Incorrect answer selected

Causes of purpura in children include:

Thrombocytopenic:

Impaired production: leukaemia, aplastic anaemia

Excessive destruction: immune (idiopathic thrombocytopenic purpura [ITP]), secondary (systemic lupus erythematosus [SLE], drugs, viral infections), alloimmune neonatal thrombocytopenia

Consumptive coagulopathy: disseminated intravascular coagulation (DIC), haemolytic uraemic syndrome, thrombotic thrombocytopenic purpura

Congenital: giant haemangioma, Wiskott-Aldrich syndrome.

Vascular disorders:

Congenital: connective tissues disorders (osteogenesis imperfecta, Ehlers-Danlos, Marfan's)

Acquired: meningococcal and other severe infections

Immune: Henoch-Schönlein purpura (HSP), connective tissue disorders (SLE).

Drugs.

Although meningococcal disease is an important differential in all children with purpura, it is not the most likely cause. Even in febrile children, only 7% will have meningococcal disease.

The platelet count in Henoch-Schonlein purpura is normal. In idiopathic thrombocytopenic purpura there is an absence of hepatosplenomegaly.

Acute leukaemia is an important differential in anybody with pancytopenia.

Microscopic haematuria plus purpura may occur in infective endocarditis, but may also be related to HSP or SLE.

Question: 37 of 100

Time taken: 17:25

Theme:Skin and Hair abnormalitiesA Alopecia areata

B Dermatitis artefacta

C Trichotillomania

D Aplasia cutis

E Sutural alopecia

F Ectodermal dysplasia

G Acrodermatitis enteropathica

H Menkes' kinky hair

I Trichorrhexis nodosa

J Telogen Effluvium

Select one option from the given list that is most suitable for the following patients:

A genetic syndrome resulting in abnormal dentition and ichthyosis.

Incorrect - The correct answer is Ectodermal dysplasia

Ectodermal dysplasia is an X linked recessive condition with features of alopecia, hypodontia and a defect in sweating. These individuals have characteristic facies.

A congenital diffuse cause of hair loss due to an abnormality of the hair shaft with a child with intractable seizures.

Incorrect - The correct answer is Menkes' kinky hair

Menkes'kinky hair syndrome is a progressive cerebral deterioration associated with seizures and twisted fractured hair. It is X linked recessive and the underlying aetiology is an abnormality of copper transport resulting in low copper and low caeruloplasmin.

A 15-year-old girl with a history of emotional difficulties presents with linear markings to her forearms and an inconsistent history.

Incorrect - The correct answer is Dermatitis artefacta

Dermatitis artefacta skin lesions are deliberately produced by patients who conceal this fact from their doctors. The condition is usually found in individuals with emotional difficulties arising from a disorder of personality.

Question: 38 of 100

Time taken: 17:46

Theme:Skin and hair abnormalitiesA Alopecia areata

B Dermatitis artefacta

C Trichotillomania

D Aplasia cutis

E Sutural alopecia

F Ectodermal dysplasia

G Acrodermatitis enteropathica

H Menkes' kinky hair

I Trichorrhexis nodosa

J Telogen effluvium

Select one option from the given list that is most suitable for the following patients.

A congenital cause for localised hair loss.

Incorrect - The correct answer is Aplasia cutis

Aplasia cutis is a localised defect with the scalp. This can be an isolated finding although it has been associated with trisomies, particularly Patau syndrome.

An acquired localised area of hair loss associated with autoimmune disease.

Incorrect - The correct answer is Alopecia areata

Alopecia areata is a local area of hair loss, usually idiopathic but sometimes associated with autoimmune (AI) diseases.

An acquired cause of hair loss resulting from emotional disturbance in a teenager.

Correct

Trichotillomania is a term for hair loss as a result of pulling of his/her own hair out, there is often a history of emotional difficulties.

Question: 39 of 100

Time taken: 18:05

Theme:Skin lesionsA Café au lait spots

B Capillary haemangioma

C Port-wine stain

D Shagreen patch

E Adenoma sebaceum

F Cutis marmorata

G Herald patch

H Perioral pigmentation

I Hypopigmentation

J Tuber

Select one option from the list that is most suitable for the following patients:

An 11-year-old boy with learning difficulties has an acne type rash over the nose.

Incorrect - The correct answer is Adenoma sebaceum

This relates to the condition known as tuberous sclerosis. It is a condition whereby hamartomatous lesions become evident in early childhood. Seizures and learning difficulties are common and skin manifestations include fibrous angiomatous lesions in the nasolabial folds known as adenoma sebaceum. White macules are seen over the trunk known as ash leaf macules and shagreen patches resemble goose-like flesh. Subungual fibromata are also common.

A 4-year-old boy with a history of focal seizures and a birth mark.

Incorrect - The correct answer is Port-wine stain

This relates to Sturge-Weber syndrome, the association and localisation of aberrant vasculature in the facial skin, eyes and meninges. It is most commonly in the trigeminal nerve distribution.

An infant with Down's syndrome has mottled appearance to the skin.

Incorrect - The correct answer is Cutis marmorata

This relates to cutis marmorata, a mottled, marbled type appearance to the skin. Common in Down's syndrome as well as other trisomies, hypothyroidism and Cornelia de Lange syndrome.

Question: 40 of 100

Time taken: 18:17

Which of the following is/are true concerning atopic eczema?

True / False Does not have a genetic basis. Incorrect answer selected

Is a generalised rash over the whole body. Incorrect answer selected

Never benefits from dietary measures. Incorrect answer selected

Should be treated in its early stages with topical corticosteroids. Incorrect answer selected

Usually starts in the first year of life. Correct

Atopic eczema often presents in childhood, affecting 15-20% of children, and there is a strong genetic basis.

Atopy is the term that describes a family predisposition to a variety of allergic conditions, including asthma, hay fever and eczema.

Dietary measures, for example exclusion of cow's milk from the diet may have a major effect on the disease course, especially in infants.

Atopic eczema usually affects the face, trunk, the backs of the knees and the front of the elbows.

The main treatments are aimed at controlling and preventing inflammation and itching and include avoiding triggers which can be environmental, such as house dust mites or common detergents, or food based, such as an allergy to dairy products, or genetic.

Frequent bathing and hydrating of the skin avoiding strong soaps, and the liberal use of moisturisers and lubricants are first line treatments.

Steroids should be used in short bursts, for acute flare-ups, and potent topical steroids are not generally recommended for long-term use, especially in children and through puberty.

Question: 41 of 100

Time taken: 18:25

Which of the following therapies is NOT appropriate for the associated condition?

(Please select 1 option) 1% hydrocortisone for infantile eczema Incorrect answer selected

Coal tar for psoriasis

Cortisone cream for alopecia areata

Permethrin for scabies

Surgical excision for a cavernous haemangioma 3 cm x 4 cm on the arm This is the correct answer

Cavernous haemangiomas are usually not present at birth but appear in the first two weeks of life. Lesions are usually on the face, neck or trunk and are well-circumscribed and lobulated.

Treatment options do not include surgical excision. Treatment may be indicated if there is inhibition of normal development - for example impairing normal binocular visual development by obstructing the vision from one eye.

It may involve systemic or local steroids, sclerosants, interferon, or laser treatment.

Alopecia areata is an autoimmune condition causing discrete areas of hair loss.

Treatment options include cortisone injections into the affected areas, and the use of topical cortisone creams.

Question: 42 of 100

Time taken: 18:38

Peeling of the palms of the hand and the soles of the feet is a characteristic feature of which of the following?

True / False Chickenpox. Incorrect answer selected

Kawasaki disease. Correct

Papular urticaria. Incorrect answer selected

Scarlet fever. Correct

Tinea infection. Correct

The secondary viraemia of varicella, featuring viral particles being spread to the skin 14-16 days after initial exposure, causes the typical vesicular rash.

Skin peeling is not a feature.

Papular urticaria is a common and often annoying disorder manifested by chronic or recurrent papules caused by a hypersensitivity reaction to the bites of mosquitoes, fleas, bedbugs, and other insects. Individual papules may surround a wheal and display a central punctum.

The histopathologic pattern in papular urticaria consists of

Mild subepidermal oedema

Extravasation of erythrocytes

Interstitial eosinophils and

Exocytosis of lymphocytes.

The reaction is thought to be caused by a haematogenously disseminated antigen deposited by an arthropod bite in a patient who is sensitive.

In Kawasaki's disease, desquamation of the fingers and toes begins in the periungual region, may involve the palms and soles, and usually is observed one to two weeks after the onset of fever.

Tinea infection is associated with peeling of the skin in the affected area, and in scarlet fever, the skin rash associated with streptococcal Group A infection, the skin begins to peel usually around the sixth day of the rash.

Question: 43 of 100

Time taken: 18:58

Theme:Cutaneous manifestations of diseaseA Koplik spots

B Erythema multiforme

C Erythema nodosum

D Dermatitis herpetiformis

E Vitiligo

F Gottron's papules

G Alopecia areata

H Mycosis fungoides

I Dermatitis artefacta

J Café au lait spots

For each of the following diseases choose the most specific mucous or cutaneous manifestation from the list of options:

Measles

Correct

Koplik spots are of pathognomonic measles found on the buccal mucosa membrane and resemble coarse granules of salt.

Dermatomyositis

Incorrect - The correct answer is Gottron's papules

Gottron's papules are pathognomonic for dermatomyositis. They are inflammatory papules found over the dorsal interphalangeal joints and they become violaceous in colour and flattened before they atrophy to leave.

Stevens-Johnson syndrome

Incorrect - The correct answer is Erythema multiforme

Hypopigmentation - Erythema multiforme major is also known as Stevens-Johnson syndrome. It is a serious systemic disease which involves the two mucous membranes plus skin. It often follows respiratory symptoms and is associated with a conjunctivitis, uveitis and bullae. It may result in fluid loss and weakness as well as anaemia and neutropenia.

Question: 44 of 100

Time taken: 19:12

Theme:Cutaneous manifestations of diseaseA Koplik spots

B Erythema multiforme

C Erythema nodosum

D Dermatitis herpetiformis

E Vitiligo

F Gottron's papules

G Alopecia areata

H Mycosis fungoides

I Dermatitis artefacta

J Café au lait spots

For each of the following diseases choose the most specific mucous or cutaneous manifestation from the list of options:

Addison's disease

Incorrect - The correct answer is Vitiligo

Vitiligo is a disorder resulting from insufficiency of melanocytes deposited in the skin. It is associated with auto-immune conditions.

Cutaneous T-cell lymphoma

Incorrect - The correct answer is Mycosis fungoides

Mycosis fungoides is acute alias T-cell lymphoma often presenting as superficial patchy dermatitis which is then developed into tumours.

Coeliac disease

Incorrect - The correct answer is Dermatitis herpetiformis

Dermatitis herpetiformis is a pruritic fascicular rash found on elbows, shoulder, buttocks and knees and is associated with coeliac disease.

Theme:Cutaneous manifestations of infectious diseaseA Rheumatic fever

B TB

C Lyme disease

D Chicken pox

E Histoplasmosis

F Cat scratch disease

G Measles

H Parvovirus

I Hepatitis B

J Herpes simplex

For each cutaneous manifestation described below choose the single most likely associated infectious disease:

Erythema infectiosum.

Incorrect - The correct answer is Parvovirus

Erythema infectiosum also known as fifth disease caused by human parvovirus, often results in a low grade temperature, slapped cheek appearance to the face and a reticular lacy rash to the arms.

Gianotti-Crosti.

Incorrect - The correct answer is Hepatitis B

Gianotti-Crosti is a syndrome of non-pruritic erythematous papules on the face, buttocks and extremities. Characteristically related to hepatitis B infection although other viruses for example Epstein-Barr virus (EBV) may be associated.

Koplik spots.

Incorrect - The correct answer is Measles

Koplik spots are white coarse granules found on the buccal mucosa opposite the back molars and are pathognomonic for measles.

Question: 46 of 100

Time taken: 20:02

Theme:Cutaneous manifestations of infectious diseaseA Rheumatic fever

B TB

C Lyme disease

D Chicken pox

E Histoplasmosis

F Cat scratch disease

G Measles

H Parvovirus

I Hepatitis B

J Herpes simplex

For each cutaneous manifestation described below choose the single most likely associated infectious disease:

Erythema nodosum in presence of abnormal chest x ray.

Incorrect - The correct answer is TB

Erythema nodosum may be associated with streptococcal reactions, rheumatic fever and tuberculosis (TB). In the presence of an abnormal chest x ray TB is the most likely answer.

Erythema marginatum

Incorrect - The correct answer is Rheumatic fever

Erythema marginatum is one of the five major criteria to make a diagnosis of rheumatic fever. It is a pink rash with pale centres and a serpiginous margin, found on the trunk and proximal limbs.

Erythema chronicum migrans.

Correct

Erythema chronicum migrans is found in Lyme disease. A febrile illness caused by Borrelia burgdorferi transmitted by bites of animal tick. The rash is characterised by red margins and central clearing.

Question: 47 of 100

Time taken: 20:15

Will the following lesions resolve spontaneously?

True / False Granuloma annulare. Correct

Large capillary cavernous haemangioma (strawberry naevus). Correct

Large umbilical granuloma. Incorrect answer selected

Molluscum contagiosum. Correct

Pyogenic granulomata. Incorrect answer selected

Granuloma annulare is a benign self limiting dermatosis associated with Crohn's disease and diabetes.

Large umbilical granuloma requires cautery with silver nitrate or tying off with a suture.

Strawberry naevus is the most common capillary malformation and is self resolving.

Pyogenic granulomata require shave, punch or laser excision and up to 50% still recur.

Question: 48 of 100

Time taken: 20:29

Which one of the following statements is true regarding scabies?

(Please select 1 option) Is best treated by salicylate emulsion. Incorrect answer selected

It can be spread by a droplet infection

It causes generalised pruritus This is the correct answer

It is caused by Staphlococcus aureus

Typically it affects the face

Scabies typically affects the interdigital webs and skin folds of the arms and legs. The itch may persist for 2-3 weeks after successful treatment.

It is caused by the mite Sarcoptes scabiei. Permethrin-containing lotions are the treatment. It is spread by skin contact and sharing clothes and bedding.

Question: 49 of 100

Time taken: 20:36

Which of the following is true of atopic eczema?

(Please select 1 option) Does not have a genetic basis Incorrect answer selected

Does not respond to dietary measures

Is a generalised rash over the whole body

Should be treated in its early stages with topical corticosteroids

Usually starts in the first year of life This is the correct answer

The typical distribution is face, ears, elbows and knees.

Cow's milk is a common cause and switching to a milk hydrolysate may assist.

It is more common in those with a family history of asthma, hay fever and eczema.

Topical steroids should be applied sparingly only if symptoms cannot be controlled.

Often it develops in the first year of life.

Question: 50 of 100

Time taken: 20:49

Are the following therapies appropriate?

True / False 1% hydrocortisone for infantile eczema. Correct

Coal tar for psoriasis. Correct

Ketoconazole for alopecia areata. Incorrect answer selected

Permethrin for scabies. Correct

Surgical excision for a cavernous haemangioma 3cm and 4cm on the arm. Incorrect answer selected

Permethrin or malathion are used for scabies (benzyl benzoate is an irritant).

Systemic or local steroids, sclerosants and laser treatments are used for cavernous haemangioma.

Steroid injections and creams are used for alopecia areata.

Question: 51 of 100

Time taken: 21:05

Erythema nodosum can be found in association with which of the following?

True / False Aspirin therapy. Incorrect answer selected

Leprosy. Correct

Miliary tuberculosis. Incorrect answer selected

Sarcoidosis. Correct

Streptococcal infection. Correct

Other causes of erythema nodosum include

Yersiniosis

Ulcerative colitis and

Sarcoidosis.

Drugs causing the condition include the oral contraceptive pill and sulphur-containing antibiotics.

Non-steroidal anti-inflammatory drugs are used in treatment.

Question: 52 of 100

Time taken: 21:17

Regarding psoriasis in children, which of the following is/are true?

True / False A common cause of alopecia. Incorrect answer selected

Associated with cow’s milk protein intolerance. Incorrect answer selected

Best treated initially with steroid creams. Incorrect answer selected

Sometimes a sequel to staphylococcal infection. Incorrect answer selected

Usually not itchy. Correct

Eczema is associated with cow's milk intolerance and itchy, rather than psoriasis, which rarely itches.

Coal tar is commonly used initially.

Small patches are found on the scalp rather than complete alopecia.

Psoriasis can occur after Strep. throat infections.

Question: 53 of 100

Time taken: 21:27

Regarding atopic eczema, which of the following statements is/are true?

True / False Is best managed with an egg and milk free diet. Incorrect answer selected

May be exacerbated by a viral infection. Correct

Predisposes to cutaneous viral infections. Correct

Usually starts within the first two weeks of life. Incorrect answer selected

Will have cleared by puberty in at least 50% of children. Correct

Cow's milk intolerance is commonly associated and soy milk may be substituted.

Atopic eczema commonly starts within the first year of life.

It predisposes to eczema herpeticum, infection with herpes simplex virus.

Infection and stress as well as environmental exposure to allergens such as dust mite will exacerbate symptoms.

Question: 54 of 100

Time taken: 21:43

Which of the following is/are true in children?

True / False All moles on the palms and soles should be excised prophylactically. Incorrect answer selected

Giant hairy moles may undergo malignant change. Correct

Malignant melanoma is very rare under the age of 10 years. Correct

Moles are usually present at birth. Incorrect answer selected

Moles can appear at any time after birth. Correct

Only 1% of moles are present at birth and termed congenital naevi.

Three percent to 10% of giant congenital moles (9 cm on head and 6 cm on the body) can give rise to melanoma.

Small moles occurring on the palms and soles may be entirely benign and should be observed for signs of change.

Moles can appear throughout life.

Question: 55 of 100

Time taken: 21:56

Are the following conditions associated with characteristic skin disorders?

True / False Cystinosis. Incorrect answer selected

Essential fatty acid deficiency. Correct

Glycogen storage disease. Incorrect answer selected

Sturge-Weber syndrome. Correct

Tuberose sclerosis. Correct

Glycogen storage disease typically affects the muscles and the liver.

Sturge-Weber is associated with capillary haemangiomas.

Adenoma sebaceum, shagreen patches and ash leaf hypopigmentation are the typical skin lesions of tuberous sclerosis.

Cystinosis commonly causes renal tubular defects.

Essential fatty acid deficiency causes a generalised scaly dermatitis, and alopecia and thrombocytopenia.

Question: 56 of 100

Time taken: 22:04

An 18-year-old male presents with small pruritic scabs in his pubic region.

What is the most likely diagnosis?

(Please select 1 option) Atopic eczema Incorrect answer selected

Lice infestation This is the correct answer

Pityrosporum folliculitis

Scabies

Tinea cruris

The diagnosis is likely to be infestation with pubic lice.

Eczema is unusual in the pubic region and the only other possibility is scabies, but there is usually no evidence of infestation except for burrows in the finger webs and generalised pruritus.

Question: 57 of 100

Time taken: 22:23

Theme:Neonatal jaundiceA Physiological jaundice

B Biliary atresia

C Hypothyroidism

D Rhesus incompatibility

E Congenital spherocytosis

F Congenital cytomegalo virus infection

G Galactosaemia

H Glucose-6-phosphate dehydrogenase deficiency

I Cystic fibrosis

J Fructose intolerance

For each of the following jaundiced babies, select the most likely cause.

A 13-day-old infant, who was noted to have an umbilical hernia and has very dry skin, presents with jaundice. She is a floppy baby. Her bilirubin is checked and is found to be elevated and mainly unconjugated. The community midwife has been unable to gain access to the home for the last week.

Incorrect - The correct answer is Hypothyroidism

Features of congenital hypothyroidism include

umbilical hernia

dry skin

hypotonia

jaundice.

This is typically unrecognisable at birth but, if not identified by screening, gives rise to

prolonged jaundice

failure to thrive

impaired growth

feeding difficulties

constipation

hypotonia.

If left untreated, even for a few weeks after birth, there will be permanent neurological damage, resulting in intellectual impairment.

A Caucasian infant has required surgery at the regional neonatal unit for meconium ileus and has developed jaundice. The serum conjugated bilirubin is 65 µmol/l. The diagnosis is eventually confirmed from the neonatal screening tests.

Incorrect - The correct answer is Cystic fibrosis

Meconium ileus and jaundice may be presenting features of CF in the neonatal period. The screening test is immuno-reactive trypsin.

A 12-day-old male baby has a conjugated and unconjugated hyperbilirubinaemia. He is breast fed and has become more uninterested in feeds. He was investigated for sepsis; blood cultures demonstrated Escherichia coli septicaemia, urine cultures were clear. His clotting is deranged.

Incorrect - The correct answer is Galactosaemia

Galactosaemia typically presents around two weeks of age with jaundice. Escherichia coli septicaemia is a feature. Treatment is by removing galactose from the diet. Cataracts are a later feature, even if treatment is instituted early.

Question: 58 of 100

Time taken: 22:45

Theme:RashesAStaphylococcal scalded skin syndrome

B Rubella

C Measles

D Kawasaki disease

E Impetigo

F Scarlatina

G Infectious mononucleosis

H Henoch-Schönlein purpura

I Meningococcal infection

J Still's disease (systemic onset juvenile chronic arthritis)

Match the following descriptions of rash with the illness for which they are the most typical exanthem.

A salmon-coloured, reticulate macular rash develops mainly over the extensor surfaces of the limbs in a 5-year-old boy with swinging temperature; hot, swollen, painful knees and left elbow and palpable spleen. The erythrocyte sedimentation rate (ESR) is 95. The blood count, C-reactive protein and chest x ray are normal.

Incorrect - The correct answer is Still's disease (systemic onset juvenile chronic arthritis)

'Salmon-coloured' is the description used to describe the rash of Still's disease. The distribution is not that of Henoch-Schönlein purpura (HSP) which covers typically the buttocks and limbs and is a purpuric rash.

A 12-year-old boy develops petechiae and papules, some of which become purpuric over his buttocks and legs, associated with painful swollen knees. There is microscopic haematuria on testing. The platelet count is normal.

Incorrect - The correct answer is Henoch-Schönlein purpura

This is a description of HSP and the well recognised complication of HSP nephritis. A proportion of these patients will develop progressive nephritis and end stage renal failure.

A 5-day-old girl has a high temperature and is irritable. She has areas of desquamation over her finger tips and in the axillae. Her carer notices that her skin blisters easily following minimal contact.

Incorrect - The correct answer is Staphylococcal scalded skin syndrome

SSSS results from infection with Staphylococci with the exofoliative toxin A and B. These exotoxins cause disruption to the epidermal layer by interfering with intercellular junctions. Mortality is up to 3% in children. The desquamation occurs concomitantly with the illness unlike Kawasaki disease and Kawasaki disease does not occur in this age group. There may be a history of minimal skin trauma which provides a port of entry for the organism.

Question: 59 of 100

Time taken: 23:06

Theme:RASHESA Staphylococcal scalded skin syndrome

B Rubella

C Measles

D Kawasaki disease

E Impetigo

F Scarlatina

G Infectious mononucleosis

H Henoch-Schonlein purpura

I Meningococcal infection

J Still's disease (systemic onset juvenile chronic arthritis)

Match the following descriptions of rash with the illness for which they are the most typical exanthem:

A 5-year-old boy has a bright red, punctate, erythematous rash which blanches on pressure, beginning in the axillae with some perioral pallor and relative facial sparing. The skin feels like "sandpaper". The rash fades and desquamates on the hands and feet. A thick white exudate develops on the tongue which peels leaving a "strawberry tongue" with prominent papillae.

Incorrect - The correct answer is Scarlatina

This description is typical of scarlatina, that is, beta haemolytic streptococcal infection. The rash may be confused with that of Kawasaki disease. However, desquamation occurs in late in Kawasaki disease.

A maculopapular rash develops in a child with sore throat and fever who has been treated with ampicillin.

Incorrect - The correct answer is Infectious mononucleosis

This is a known effect of giving ampicillin during Epstein-Barr virus (EBV) infection.

A 3-year-old child presents with high fever for seven days, conjunctival infection, fissuring of the lips and strawberry tongue, erythema followed by desquamation of the hands and feet, and a macular rash over the trunk with cervical lymphadenopathy.

Incorrect - The correct answer is Kawasaki disease

These are major and minor features of Kawasaki disease. High fever and desquamation are typical.

Question: 60 of 100

Time taken: 23:18

A 3-year-old boy presents with a rash on the face.

It had appeared two days after he accidentally scratched his face. The rash has been worsening over the past three to four days. A similar rash appeared yesterday on his trunk.

He was born at 39/40 weighing 2.97 kg and there were no neonatal problems. He is fully immunised, on no medication, and there is no family history of note.

On examination his temperature is 36.5°C (tympanic), RR 15/min and HR 95/min. He has eight raised lesions 2-5 mm across around his mouth and three more on his trunk. They have a golden crust. He has enlarged glands in the neck.

What is the most likely diagnosis?

(Please select 1 option) Contact dermatitis Incorrect answer selected

Erythema multiforme

Impetigo This is the correct answer

Stevens-Johnson syndrome

Viral exanthems

The history of golden crusted lesions in this distribution suggests non-bullous impetigo. This is often auto-inoculated to other parts of the body via the fingernails. Regional lymph nodes are often enlarged.

In bullous impetigo, flaccid bullae develop which may be considerably larger.

Treatment is with oral antibiotics, particularly to cover Staph. aureus.

Question: 61 of 100

Time taken: 23:26

A 10-month-old girl is referred with a florid skin rash.

Soon after birth she developed cradle cap and dry skin on the cheeks. This gradually improved, but over the past few months she has had dry red skin over the trunk, the elbows, wrists, knees, ankles and base of the ears. This is itchy and disturbs sleep.

She was born at 37+3/40 weighing 3.34 kg and there were no other neonatal problems. She has been fully immunised. Mother has hay fever and asthma.

On examination she is on the 10th centile for height, weight and OFC. Temperature is 36.4°C, RR 30/min and HR 100/min. She has a dry red scaly rash on patches of the trunk, elbows, wrists, ankles and knees. She has enlarged glands in the axillae and groins.

What is the most likely diagnosis?

(Please select 1 option) Atopic dermatitis Correct

Contact dermatitis

Erythema multiforme

Impetigo

Stevens-Johnson syndrome

The type, distribution and progress of the rash, in conjunction with the family history, suggest atopic excema.

This is treated with moisturisation and topical anti-inflammatories (steroids), the strength of the latter being titrated against effect.

Irritants and dessicants should be avoided.

Question: 62 of 100

Time taken: 23:40

A 12-year-old boy presents with a florid rash on the hands and feet.

He became ill three days before, when he developed painful mouth ulcers. Yesterday he developed a rash on the hands and feet.

He was a full term normal delivery, and previously has been very healthy. He is on no medications, is fully immunised, and there is no family history of note.

On examination he has a temperature of 38.5°C, RR 15/min and HR 85/min. He has profuse oral ulcers on his tongue and buccal mucous membrances. He has many circular raised pinky red lesions over the hands and feet with darker centres. He has tender cervical lymphadenopathy.

What is the most likely diagnosis?

(Please select 1 option) Contact dermatitis Incorrect answer selected

Erythema multiforme This is the correct answer

Impetigo

Stevens-Johnson syndrome

Viral exanthems

The history suggests an attack of oral ulceration, followed by the development of target lesions over the extremities.

The likely diagnosis is primary herpes simplex type 1 infection, complicated by erythema multiforme.

Treatment is supportive, though acyclovir may be given to control the herpes.

Question: 63 of 100

Time taken: 23:47

A 5-month-old boy presents with florid red rash over his scalp, nappy area and trunk. Despite this he seems well in himself and feeding well.

He was born at term weighing 3.1 kg and there were no neonatal problems. He is fully immunised and there is no FH/SH of note.

On examination he is apyrexial and well. The rash is florid red and confluent over his nappy area. He has a crusty confluent covering over his scalp, extending onto his forehead. Smaller 0.5-1 cm greasy lesions are present over his trunk.

What is the most likely diagnosis?

(Please select 1 option) Contact dermatitis Incorrect answer selected

Erythema multiforme

Impetigo

Seborrhoeic dermatitis This is the correct answer

Stevens-Johnson syndrome

The picture is of extensive greasy scaly rash especially over the head (cradle cap) and nappy area without systemic upset.

This is highly characteristic of seborrhoeic dermatitis.

Selenium shampoo and topical steroids usually result in rapid resolution.

Question: 64 of 100

Time taken: 23:54

A 7-month-old girl presents with fever and a rash.

She was completely well till five days ago, when she developed a slight cold. The next day she developed fever to 39.7°C, which has persisted despite antipyretics.

Despite this she has remained relatively well and continues to drink, though her appetite is poor. Today she has developed a rash over the face and trunk.

She was born at term weighing 3.8 kg and there were no neonatal problems. She is fully immunised to date and there is no FH/SH of note.

On examination she has a temperature of 36.8°C, RR 25/min and HR 100/min. The rash is macular, profuse, pink and blanching. It is most prominent over the face and trunk. She has shotty cervical lymphadenopathy.

What is the most likely diagnosis?

(Please select 1 option) Contact dermatitis Incorrect answer selected

Erythema multiforme

Impetigo

Stevens-Johnson syndrome

Viral exanthems This is the correct answer

The history of a well child with high fever for a few days followed by resolution of fever at around the time of appearance of a rose-coloured rash is characteristic of roseola infantum.

Since the introduction of measles, mumps, rubella vaccine (MMR), this is by far the commonest cause of a measles-like rash.

The peak incidence is 6-18 months. 5% develop febrile seizures.

It is caused by human herpes virus 6 and 7.

Question: 65 of 100

Time taken: 24:04

A 13-year-old boy is brought to clinic by his parents who are concerned about his weight.

Which of the following is true of childhood obesity?

(Please select 1 option) It can lead to overestimation of the dose of intravenous fluids when these are required Correct

It is defined as a body mass index above 28 kg/m2

It is more common in families in social class I than in social class V

It is unlikely to lead to adult obesity

It is usually associated with hypogonadism

Obesity is defined as a BMI greater than 30 kg/m2.

Although hypogonadism may be a feature of disorders such as Prader-Willi syndrome, hypothalamic pituitary testicular function is usually unimpaired in childhood obesity.

Obese children frequentl go on to become obese adults with increased prevalence amongst the lower social classes.

It may lead to overestimation of the dose of an intravenous drug as the doses are based on body weight assuming a substantial proportion of that is lean tissue (hydrous) as opposed to fat (anhydrous).

As a consequence of the metabolic activity of the fat tissue above average height before puberty is a feature.

Question: 66 of 100

Time taken: 24:12

Which of the following is/are true of childhood obesity?

True / False It can lead to overestimation of the dose of intravenous fluids when these are required Correct

It is more common in families in social class I than in social class V Incorrect answer selected

It is unlikely to lead to adult obesity Incorrect answer selected

It is usually associated with above average height before puberty Correct

It is usually associated with hypogonadism Incorrect answer selected

Obesity is defined as a BMI greater than 30 kg/m2.

Although hypogonadism may be feature of disorders such as Prader-Willi syndrome, hypothalamic pituitary testicular function is usually unimpaired in childhood obesity.

Obese children frequently go on to become obese adults with increased prevalence amongst the lower social classes.

It may lead to overestimation of the dose of an intravenous drug as the doses are based on body weight assuming a substantial proportion of that is lean tissue (hydrous) as opposed to fat (anhydrous).

As a consequence of the metabolic activity of the fat tissue above average height before puberty is a feature.

Question: 67 of 100

Time taken: 24:24

A 3-month-old infant boy is brought to you by his mother who is concerned about a rash on his body.

Which of the following supports a diagnosis of eczema?

(Please select 1 option) Cold weather relieves the symptoms Incorrect answer selected

Dermographism excludes the diagnosis

The rash has been present from birth

The rash is fluctuant

The rash is pruritic This is the correct answer

It is unusual for the rash of infantile eczema to be present at birth and usually occurs around 3 months after birth.

Pruritus is a typical feature and relief with warmer conditions may occur.

Dermographism is also a feature.

The rash tends to be fluctuant and usually resolves by aged 10.

Treatment is usually with emollients, though steroid creams may be required for more severe cases.

A family history of atopy, hay fever, asthma or eczema is found in the vast majority of cases.

Question: 68 of 100

Time taken: 24:37

Which of the following is/are true in infantile eczema?

True / False A family history of atopy is elicited in 70% of cases Correct

Cold weather relieves the symptoms Incorrect answer selected

Dermographism excludes the diagnosis Incorrect answer selected

The papules are itchy Correct

The rash is characteristically present at birth Incorrect answer selected

It is unusual for the rash to be present at birth and usually occurs weeks after birth.

Pruritus is a typical feature and relief with warmer conditions may occur.

Dermographism is also a feature.

A family history of atopy, hay fever, asthma or eczema is found in the vast majority of cases.

Question: 69 of 100

Time taken: 24:49

Can the following conditions present in a newborn infant as a bullous eruption?

True / False Atopic eczema Incorrect answer selected

Epidermolysis bullosa Correct

Mastocytosis Correct

Phenylketonuria Incorrect answer selected

Syphilis Correct

So-called syphilitic pemphigus may present with a bullous eruption.

Bullous eruptions are a feature of mastocytosis, a condition associated with proliferation of mast cells in skin (cutaneous) or systemically.

Cutaneous mastocytosis is commonest and is associated with epidermolysis bullosa.

Neither atopic eczema (unlike contact dermatitis) nor phenylketonuria (associated with eczema) are associated with bullous eruptions.

Question: 70 of 100

Time taken: 24:54

Which of the following suggests a diagnosis of molluscum contagiosum rather than chickenpox?

(Please select 1 option) Absence of erythema surrounding lesions Correct

Lesions disappearing within a month

Positive contact history

Presence of macules and papules

Presence of pruritis

Molluscum contagiosum is caused by a DNA pox virus.

The lesions are small, skin coloured papules with central umbilication. There is little surrounding inflammation and they may be spread following scratching to other sites.

Chickenpox lesions in the early stages may be mistaken for molluscum. However, the presence of associated macules and later vesicles and pustules help to differentiate them. These lesions also affect the mucus membranes, and usually disappear within a few weeks, while molluscum can persist for up to a year.

Copyright © 2002 Dr Colin Melville

Question: 71 of 100

Time taken: 25:06

In pustular psoriasis, which of the following statements is/are true/false?

True / False It usually responds to topical therapy. Incorrect answer selected

Lesions are confined to the soles of the feet. Incorrect answer selected

The mucous membranes are generally affected. Incorrect answer selected

Onset may follow a Streptococcal respiratory infection. Correct

Pustules are sterile. Correct

Pustular psoriasis may be a severe form of the disease, and may be either localised of generalised.

In its most severe form, there is psoriatic erythroderma affecting the whole body. This may require hospitalisation and systemic treatment.

Involvement of the mucous membranes may suggest Reiter's syndrome, since psoriasis does not affect them.

Copyright © 2002 Dr Colin Melville

Question: 72 of 100

Time taken: 25:17

A 3-month-old baby presents with a nappy rash.

Is it true that the following should be considered?

True / False Histiocytosis X Correct

Insect bites Incorrect answer selected

Scabies Incorrect answer selected

Seborrhoeic dermatitis Correct

Wiskott-Aldrich syndrome Correct

Napkin rashes are common, though they have become rarer since the advent of disposable nappies.

Previously, poor hygiene, with chemical irritation, was the commonest cause, often with supra-added candidiasis.

Seborrhoeic dermatitis and atopic eczema can also present in this way, the latter being associated with an uncomfortable child.

Rare causes include:

Acrodermatitis enteropathica

The Letterer-Siwe form of Langerhan's cell histiocytosis and

Wiskott-Aldrich syndrome.

Infants with scabies tend to have lesions on the palms, soles and trunk, and occasionally the head and neck area.

Copyright © 2002 Dr Colin Melville

Question: 73 of 100

Time taken: 25:28

A 6-year-old girl is noted to have café-au-lait spots.

Should the following diagnoses be considered?

True / False Ataxia telangiectasia Correct

Fabry's disease Incorrect answer selected

McCune-Albright syndrome Correct

Noonan's syndrome Incorrect answer selected

Xeroderma pigmentosum Incorrect answer selected

Café-au-lait spots are uniformly hyperpigmented, sharply demarcated, macular lesions. One to three spots are common in normal children, and they may be present at birth or develop during childhood.

Large asymmetric lesions are characteristic of McCune-Albright's syndrome (polyostotic fibrous dysplasia, precocious puberty, hyperfunctional endocrinopathies).

They are also characteristic of neurofibromatosis type I and are found in type II as well. In the former, in the pre-pubertal child there are five or more spots of more than 5 mm, while in post-pubertal there are more than six spots of more than 10 mm.

Café-au-lait spots are also found in:

Russell-Silver syndrome

Ataxia telangiectasia

Fanconi anaemia

Bloom's syndrome (increased chromosomal fragility)

Tuberous sclerosis

Gaucher's disease

Chediak-Higashi syndrome

Multiple lentigines

Epidermal naevus syndrome.

Copyright © 2002 Dr Colin Melville

Question: 74 of 100

Time taken: 25:41

Are the following characteristic of scabies?

True / False Bullous rash. Incorrect answer selected

Easy discrimination from eczema. Incorrect answer selected

Involvement of the palms, soles and trunk in infants. Correct

Severe itching. Correct

Secondary bacterial infection. Correct

Scabies is caused by Sarcoptes scabiei infestation.

The female burrows into the stratum corneum with the release of toxic or antigenic substances. The extent and duration of physical contact with an affected individual determines the spread. The isolated mite dies in two to three days.

There is intense pruritis with 1-2 mm red papules, which become excoriated crusted or scaling.

Thread-like burrows may not be seen in infants, in whom bullae and pustules are relatively common.

The lesions may be difficult to distinguish from eczema.

Palms, soles, face and scalp are often affected in infants, while in older children, the pattern resembles adults (interdigital, wrists, ankles, groin).

Untreated, secondary infection may occur.

Bullae are a very rare presentation and therefore not characteristic.

Copyright © 2002 Dr Colin Melville

Question: 75 of 100

Time taken: 25:53

Regarding a 12-year-old girl who presents with a hypopigmented sharply circumscribed 5 cm x 5 cm macule on the face, which of the following statements is/are true/false?

True / False She may have an increased risk of diabetes mellitus. Correct

There is spontaneous repigmentation in 50% of patients. Incorrect answer selected

There may be a history of trauma. Correct

There may be an association with multiple endocrine neoplasias. Incorrect answer selected

There may be a positive family history in 40%. Correct

Vitiligo is an acquired pigmentary defect, with 50% of cases presenting before the age of 20 years.

The lesions are sharply circumscribed depigmented macules that vary in size and shape.

There is a family history in 40%, and there may be associated uveitis and premature greying of the hair.

There is an association with autoimmune diseases including:

Hypo- or hyperthyroidism

Adrenal insufficiency

Pernicious anaemia

Diabetes mellitus.

The distribution of involvement is generally symmetric, but is occasionally unilateral or dermatomal and is commoner in sites of repeated trauma.

There is spontaneous repigmentation in 10-20% of patients.

Oral or topical psoralens, compound UV light, or high potency topical steroids may sometimes repigment small areas. Otherwise, specially prepared make-up can camouflage small lesions.

Exposure to sunlight should be avoided using sunscreen.

Copyright © 2002 Dr Colin Melville

Question: 76 of 100

Time taken: 26:03

Do the following suggest a diagnosis of seborrhoeic dermatitis rather than atopic eczema of infancy?

True / False Contiguous involvement of the scalp, ear and eyebrows with greasy brown scales. Correct

Intense pruritis. Incorrect answer selected

Lesions predominantly on the cheeks. Incorrect answer selected

Presence of associated petechiae. Incorrect answer selected

Weeping crusted lesions on the scalp. Incorrect answer selected

In seborrhoeic dermatitis, lesions begin particularly on the scalp (cradle cap) involving the ear and contiguous skin of the nose, eyebrows and eyelids with greasy brown scales.

In contrast, in infantile atopic dermatitis, there is erythema, weeping and crusting of the lesions, with associated pruritis.

Atopic dermatitis often begins on the cheeks and does not involve the palms and soles, which would suggest scabies.

Seborrhoeic dermatitis usually responds more rapidly to treatment.

The presence of petechiae would suggest eczema with associated thrombocytopenia and Wiskott-Aldrich syndrome should be excluded.

Copyright © 2002 Dr Colin Melville

Question: 77 of 100

Time taken: 26:16

Regarding giant congenital melanocytic naevi, which of the following statements is/are true/false?

True / False Account for 3% of all melanomas. Correct

Are between 5 cm and 10 cm in diameter. Incorrect answer selected

Involve only the skin. Incorrect answer selected

Most often affect the limbs. Incorrect answer selected

Occur in 1 in 2000 live births. Incorrect answer selected

Congenital melanocytic naevi are present in 1% of newborn infants.

Giant means greater than 20 cm, small less than 2 cm and intermediate between these.

They should be distinguished from:

Mongolian blue spots

Café-au-lait spots

Smooth muscle hamartomas

Dermal melanocytosis (naevi of Ota and Ito).

Small naevi affect the lower trunk, upper back, shoulders, chest and proximal limbs. They vary from brown to blue or black.

About 10% of melanomas arise from small congenital naevi.

Giant congenital naevi occur in fewer than one in 20,000 births, particularly on the posterior trunk, but are also seen on the head or the limbs. They are associated with leptomeningeal melanocytosis, and have a predisposition for the development of malignant melanoma.

They may cause:

Raised intracranial pressure

Hydrocephalus

Seizures

Mental retardation.

Central nervous system (CNS) involvement may be asymptomatic and found only on MRI scan.

Approximately half of all melanomas that do occur within giant naevi occur by 5 years of age.

The overall incidence is 5-10% with approximately 3% of all melanomas arising within congenital giant naevi.

Copyright © 2002 Dr Colin Melville

Question: 78 of 100

Time taken: 26:29

Regarding eczema herpeticum, which of the following statements is/are true?

True / False Is invariably fatal if untreated. Incorrect answer selected

Is more severe in reactivation disease. Incorrect answer selected

Is typically associated with a high fever for over a week. Correct

Only a single crop of vesicles usually appear. Incorrect answer selected

Usually has an indolent onset. Incorrect answer selected

Eczema herpeticum is the result of primary infection of eczematous skin with herpes simplex virus (HSV). The severity varies from mild to fatal.

There is usually an abrupt onset with crops appearing over seven to nine days. These may become coalesced.

Typically, the child has a high fever for seven days, and recurrent attacks can occur.

Death can result from:

Physiological disturbances (loss of fluid electrolytes and protein through the skin) or

Dissemination of the virus to brain and other organs or

Secondary bacterial sepsis.

Copyright © 2002 Dr Colin Melville

Question: 79 of 100

Time taken: 26:41

Which of the following are recognised causes of anterior uveitis?

True / False Herpetic keratitis Correct

Kawasaki disease Correct

Pauciarticular rheumatoid arthritis Correct

Sarcoidosis Correct

Trauma Correct

The uveal tract consists of the inner vascular coat of the eye (iris, ciliary body, choroid).

Inflammation results from both infectious and non-infectious causes, including a number of systemic diseases and trauma. Either part or the whole of the uveal tract may be preferentially involved.

Iritis may be caused by:

Pauciarticular rheumatoid arthritis

Kawasaki disease

Sarcoidosis, secondary to corneal disease (herpetic keratitis, bacterial or fungal corneal ulcer, corneal abrasion, foreign body)

Traumatic iritis of iridocyclitis.

Choroiditis is inflammation of the posterior portion of the uveal tract, and invariably involves the retina (choreoretinitis).

Causes include:

Toxoplasmosis

Histoplasmosis

Cytomegalovirus (CMV)

Sarcoid

Syphilis

Tuberculosis (TB)

Toxocariasis.

Secondary complications include retinal detachment, glaucoma, phthisis.

Copyright © 2002 Dr Colin Melville

Question: 80 of 100

Time taken: 26:53

A 2-year-old boy presents with generalised epilepsy.

If this were due to tuberous sclerosis, might the following clinical findings be expected?

True / False Atrial fibrillation due to cardiac rhabdomyoma Correct

Developmental delay Correct

Periungual fibromas Incorrect answer selected

Positive family history of epilepsy Correct

Presence of ash-leaf macules Correct

Tuberous sclerosis is a multisystem disorder affecting ectodermal tissues, but also involving the eye, kidney and heart (mesodermal/endodermal).

The classical triad is skin lesions in association with epilepsy and mental retardation.

It is autosomal dominant, but with very variable expression.

The earliest sign is the ash-leaf macule which is present at birth or early infancy (also present in two to three per 1000 normal newborns). These are 0.5-3 cm lesions shaped like a mountain ash leaf.

Adenoma sebaceum and periungual fibromata appear in later childhood or puberty.

Café-au-lait spots are less numerous than in neurofibromatosis.

Seventy percent have mental deficiency, nearly all of whom have epilepsy. Epilepsy is present in 70% of those patients without MR.

Other tumours include:

Phakomata (flat grey lesions on the retina)

Malignant astrocytoma

Rhabdomyoma (heart)

Renal hamartoma.

Copyright © 2002 Dr Colin Melville

Question: 81 of 100

Time taken: 27:03

A 10-year-old girl presents with an overdose of Paracetamol:

True / False Acetylcystine should routinely be given if the presentation is within the first 12 hours of overdose. Incorrect answer selected

Because she is over the age of 6, she is unlikely to develop significant toxicity. Incorrect answer selected

Liver function tests should be monitored. Correct

The mortality in those with an AST of >350/IU/L is 4%. Incorrect answer selected

Hospitalisation will be needed for at least 5 days. Incorrect answer selected

Treatment with N-acetyl cysteine (NAC) is given according to a standard nomogram. NAC may be useful up to 36 hours following ingestion. Children under the age of 6 are unlikely to develop significant toxicity, but adolescents have a higher incidence of toxic plasma levels following ingestion, and a higher incidence of abnormal AST >1000/U/L. Even after serious hepatotoxicity, the mortality rate is under 0.5%. The occasional patient may require liver transplantation.

Copyright © 2002 Dr Colin Melville

Question: 82 of 100

Time taken: 27:16

Which of the following suggest/s a diagnosis of molluscum contagiosum rather than chickenpox?

True / False Absence of erythema surrounding lesions Correct

Lesions disappearing within a month Incorrect answer selected

Positive contact history Incorrect answer selected

Presence of macules and papules Incorrect answer selected

Presence of pruritis Incorrect answer selected

Molluscum contagiosum is caused by a DNA pox virus.

The lesions are small, skin coloured papules with central umbilication. There is little surrounding inflammation and they may be spread following scratching to other sites.

Chickenpox lesions in the early stages may be mistaken for molluscum. However, the presence of associated macules and later vesicles and pustules help to differentiate them.

These lesions also affect the mucus membranes, and usually disappear within a few weeks, while molluscum can persist for up to a year.

Copyright © 2002 Dr Colin Melville

Question: 83 of 100

Time taken: 27:27

Is it true/false that generalised pruritis can occur inthe following?

True / False Acute renal failure Incorrect answer selected

Chronic hepatic failure Correct

Hodgkin's lymphoma Correct

Secondary polycythaemia Incorrect answer selected

The later stage of pregnancy Correct

Local causes:

Tapeworms

Allergic rhinitis

Lichen simplex

Vulvitis (including gonococcal)

Amoebiasis

Hookworm.

Generalised causes:

Atopic dermatitis

Urticaria

Serum sickness

Late pregnancy

Parvovirus

Drugs

Vitamin A toxicity

Conjugated jaundice (due to retained bile acids and improved by cholestyramine)

Hodgkin's disease

Renal osteodystrophy

A variety of dermatological abnormalities.

Copyright © 2002 Dr Colin Melville

Question: 84 of 100

Time taken: 36:43

Are the following seen in Crohn's disease?

True / False Dermatitis herpetiformis Incorrect answer selected

Erythema nodosum Correct

Haemolytic anaemia following sulphasalazine treatment Correct

Pyoderma gangrenosum Correct

Steatorrhoea Correct

Dermatitis herpetiformis is a manifestation of coeliac disease.

Fat malabsorption may be a feature of Crohn's with erythema nodosum and pyoderma gangrenosum being cutaneous manifestations. Oligoarthritis with sacroilitis and iritis may also feature.

Sulphasalazine can produce a number of haematological problems in particular thrombocytopenia and leucopenia but haemolysis can occur.

Question: 85 of 100

Time taken: 36:59

Do recognised complications of erythroderma (generalised exfoliative dermatitis) include the following?

True / False Disturbances of pigmentation Correct

High output cardiac failure Correct

Hyperpyrexia Correct

Hypothermia Correct

Multiple squamous carcinomata of skin Incorrect answer selected

Erythroderma is a medical emergency and is most commonly due to psoriasis or extensive dermatitis but other causes include

Drug eruption

Sezary syndrome

Pityriasis rubra pilaris.

Typical features may include

Pyrexia and hypothermia (due to the heat loss)

Infection and

Dehydration.

Other features include

Lymphadenopathy

Peripheral oedema

Hypoalbuminaemia.

Question: 86 of 100

Time taken: 37:13

Which of the following dermatological disorders is correctly matched to its treatment?

(Please select 1 option) Acne and steroids Incorrect answer selected

Erythema nodosum and topical tetracycline

Lipoma and laser therapy

Psoriasis and vitamin D analogues This is the correct answer

Stevens-Johnson syndrome and retinoids

Stevens-Johnson syndrome is a condition associated with erythema multiforme and ulceration of the mucous membranes. Treatment includes steroids and treating the underlying disorder.

Erythema nodosum is appropriately treated through treating the underlying condition - causes include sarcoid, mycoplasma pneumonia and drugs.

Sebaceous cysts, lipomas and dermatofibromas are appropriately treated with surgery and do not respond to topical therapies.

Question: 87 of 100

Time taken: 37:29

Theme:Features of herpetic infectionsA Chicken pox

B Eczema herpeticum

C Erythema multiforme

D Facial palsy

E Genital ulcers

F Gingivostomatitis

G Herpetic whitlow

H Keratitis

I Neuralgia

J Zoster

For each of the following patients below, choose the single most likely herpetic feature from the above list of options. Each option may be used once, more than once or not at all.

An 8-year-old boy has been unwell with fever and malaise for the past two days. He has recently developed a rash that is mainly on the face and trunk. The rash consists of papules, vesicles and crusted lesions.

Correct

Chicken pox is distributed centripetally with the trunk affected more than the extremities. The individual lesions are in various stages of development (macules, papules, vesicles, crusted and healed lesions). The lesions of small pox all present in the same stage and the limbs are more affected than the trunk.

A 14-year-old girl presents with an asymptomatic erythematous annular rash more obvious on the extremities. She also has a cold sore.

Incorrect - The correct answer is Erythema multiforme

Erythema multiforme presents with annular lesions, sometimes bullous and more common on the extremities. The commonest cause is infection with herpes simplex virus. Other infections that may cause it are vaccinia, infectious mononucleosis, hepatitis, mycoplasma and histoplasmosis.

Question: 88 of 100

Time taken: 37:43

Vesicles are characteristic in which of the following?

True / False Dermatitis herpetiformis Correct

Erythema nodosum Incorrect answer selected

Exfoliative dermatitis Incorrect answer selected

Porphyria cutanea tarda Correct

Stevens-Johnson syndrome Correct

Vesicles occur in:

Erythema multiforme (Stevens-Johnson)

Chicken pox

Herpes zoster

Porphyria cutanea tarda

Dermatitis herpetiformis

Pemphigus/pemphigoid.

They do not occur in erythema nodosum which has raised tender nodules usually on the shins nor in exfoliative dermatitis.

Question: 89 of 100

Time taken: 37:56

Molluscum contagiosum

True / False is a DNA virus of the pox family Correct

is found only in humans Correct

produces severely pruritic lesions Incorrect answer selected

produces characteristic paracentric umbilication of the papules Correct

treatment involves leaving lesions alone to resolve spontaneously Correct

Molluscum contagiosum is a superficial skin infection. The virus invades the skin causing the appearance of firm, flesh-colored, doughnut-shaped bumps, about 2-5 mm in diameter which are usually non-pruritic. Molluscum contagiosum is caused by a virus belonging to the poxvirus family. Close physical contact is usually necessary for transmission; indirect transmission from shared towels, swimming pools, etc., may also be responsible for infection. The incubation period varies from several weeks to several months. Treatment can be with surgical excision, cryotherapy, trichloroacetic acid or cantharadin topical therapies. However, the lesions are best left alone and will usually resolve spontaneously

Question: 90 of 100

Time taken: 38:14

Molluscum contagiosum

True / False is a DNA virus of the pox family Correct

is found only in humans Correct

produces severely pruritic lesions Incorrect answer selected

produces characteristic paracentric umbilication of the papules Correct

treatment involves leaving lesions alone to resolve spontaneously Correct

Molluscum contagiosum is a superficial skin infection. The virus invades the skin causing the appearance of firm, flesh-colored, doughnut-shaped bumps, about 2-5 mm in diameter which are usually non-pruritic. Molluscum contagiosum is caused by a virus belonging to the poxvirus family. Close physical contact is usually necessary for transmission; indirect transmission from shared towels, swimming pools, etc., may also be responsible for infection. The incubation period varies from several weeks to several months. Treatment can be with surgical excision, cryotherapy, trichloroacetic acid or cantharadin topical therapies. However, the lesions are best left alone and will usually resolve spontaneously

Question: 91 of 100

Time taken: 38:28

Which of the following statements regarding napkin rashes are correct?

True / False Always caused by ammonia. Incorrect answer selected

Less common in babies fed a soy protein milk. Incorrect answer selected

Often successfully treated by exposure to air. Correct

Evidence of early infantile eczema. Incorrect answer selected

More common in boys. Incorrect answer selected

Causes of napkin rash include contact dermatitis, which may produce ammonia and this may burn the skin. Infection with bacteria and candida yeasts may cause nappy rash, as can psoriasis and atopic dermatitis affecting the nappy area. Nappy rash is not an indicator of infantile eczema, and it is not more common in boys, nor is it less common in soy fed infants. Treatment is best achieved by prevention by frequent (disposable absorbent) nappy changing, and fluid feeding early in the day to lessen night time urination. Anti-fungal and Lotrimin lotions may also be useful.

Question: 92 of 100

Time taken: 38:41

A 14-year-old girl presents with severe nodulocystic acne that has not responded to conventional therapy. On commencing isotretinoin therapy, the following advice should be given:

True / False Pregnancy should be avoided using the progesterone-only contraceptive pill. Incorrect answer selected

Inflammation of the eyes and lips may occur. Correct

Increased hair growth may be a problem. Incorrect answer selected

It is most effective if used together with Tetracycline. Incorrect answer selected

Exposure to sunlight will increase its effectiveness. Incorrect answer selected

Isotretinoin is indicated for moderate to severe acne, particularly associated if associated with severe psychological disturbance. Standard course is 4-5 months. After this 30% are cured, 35% need

conventional topical and/or oral medications, and 25% have relapses requiring a further course. Side effects include:

Teratogenicity: it is contraindicated in pregnancy. The progesterone-only pill is insufficient precaution on its own.

Inflammation: chelitis, cirrhosis, blepharoconjunctivitis, periodic epistaxis.

Raised triglyceride and cholesterol levels (check LFTs pre-therapy).

Arthralgia, myalgia, depression, hair thinning, photosensitivity, Staphylococcal infections.

Tetracycline should not be used concurrently, as this may precipitate benign intracranial hypertension.

Copyright © 2002 Dr Colin Melville

Question: 93 of 100

Time taken: 38:53

Retinitis pigmentosa is associated with:

True / False Progressive loss of peripheral vision Correct

Friedreich's ataxia Incorrect answer selected

Usher's Syndrome Correct

Cataract Incorrect answer selected

Abetalipoproteinaemia Correct

In retinitis pigmentosa there is retinal degeneration associated with pigmentary changes, arteriolar attenuation, optic atrophy, and progressive impairment of visual function. The appearance ranges from granular or mottling to focal pigment aggregates with bone spicules. Initially, there is loss of night vision, followed by progressive loss of peripheral vision. The retinal function (as measured by ERG) is reduced. The disorder may be AR, AD or X-linked. A special form is Leber congenital retinal amaurosis in which visual impairment is evident soon after birth. Similar secondary changes are seen in a wide range of metabolic and neurogenerative diseases including:

Mucopolysaccharidoses.

Late onset gangliosidoses.

Abetalipoproteinaemia.

Kearns-Sayre Syndrome (progressive external ophthalmoplegia).

Laurence-Moon-Biedl Syndrome.

Usher Syndrome (with hearing loss).

Copyright © 2002 Dr Colin Melville

Question: 94 of 100

Time taken: 39:06

The following are characteristic of psoriasis:

True / False Lesions affecting the eyebrows. Incorrect answer selected

Involvement of the mucous membranes. Incorrect answer selected

Episodes precipitated by Streptococcal infections. Correct

Profuse small oval or round lesions on the trunk. Correct

Positive Koebner phenomenon. Correct

Psoriasis is a common skin disorder, affecting particularly girls, and in 50% there is a positive family history. There is an association with HLA-DW6, and epidermal turnover time is accelerated. Characteristic are plaques which are thick and silvery, removal resulting in pinpoint bleeding. New lesions appear at sites of trauma (Koebner phenomenon). Lesions occur anywhere but particularly the scalp, knees, elbows, umbilicus, and genitalia. Pitting of the nail plate and detachment of the nail plate may occur. Specific forms include:

Psoriatic erythroderma.

Localised or generalised pustular psoriasis.

Linear psoriasis.

Guttate psoriasis (droplet like over the trunk).

Streptococcal infections in which the organism produces exotoxins acting as superantigens, may cause the disease in a genetically susceptible host.Copyright © 2002 Dr Colin Melville

Question: 95 of 100

Time taken: 39:17

An 18 month old girl presents with an itchy rash. The following conditions should be considered:

True / False Seborrhoeic dermatitis Incorrect answer selected

Pityriasis versicolor Incorrect answer selected

Atopic eczema Correct

Scabies Correct

Dermatitis herpetiformis Correct

Itching is an extremely useful symptom is distinguishing rashes. It is caused by:

Atopic eczema, urticaria, contact dermatitis, dermatitis herpetiformis, pityriasis rosea.

Chickenpox, scabies, insect bites, fungal infections.

Seborrhoeic dermatitis presents in the first 2 months of life with an erythematous scaly scalp eruption. Fortunately, the child remains very well and unbothered by it. In pityriasis the pale patches affect abdomen, chest and neck, and may be difficult to see in fair-skinned unless tanned. They may itch slightly, but this is not a prominent feature. Copyright © 2002 Dr Colin Melville

Question: 96 of 100

Time taken: 39:29

In a 6-year-old boy with erythema nodusum, the following should be considered:

True / False Reiter's Disease Incorrect answer selected

Ulcerative colitis Correct

Cytomegalovirus infection Incorrect answer selected

Toxoplasmosis Correct

Kawasaki Disease Incorrect answer selected

Erythema nodosum is characterised by painful, indurated, shiny, red, hot, elevated nodules 1-3cm diameter particularly on the shins. There may be associated fever, malaise, and arthralgia ?? hilar

adenopathy. Over a period of days they become violaceous, then dull purple then fade like a large bruise without residual ulceration or scar. There may be crops over 3-6 weeks. They are uncommon under the age of 6, and are commoner in females than males. Causes include:

INFECTIONS:

bacteria: Streptococci, leptospirosis, cat-scratch disease, psittacosis, yersinia.

viruses: EBV.

OTHER:

TB, tularaemia, histoplasmosis, toxoplasmosis, coccidiodomycosis.

DRUGS:

sulphonamides, oral contraceptive pill.

SYSTEMIC DISEASES:

SLE, vasculitis, regional enteritis, ulcerative colitis, Behcet's Syndrome.

Copyright © 2002 Dr Colin Melville

Question: 97 of 100

Time taken: 39:40

A 15-year-old boy was treated with permethrin cream for scabies infestation.

On follow-up three weeks later, he was found to have continuing infestation.

What is the most likely reason for this?

(Please select 1 option) Facial skin was not treated Incorrect answer selected

Non-disposal of underwear

Other household members were not treated This is the correct answer

The organism is resistant to permethrin

The treatment was not repeated as prescribed

Scabies is an intensely pruritic and highly contagious infestation of the skin, acquired through close personal contact.

A delayed type IV hypersensitivity reaction to the mites, their eggs, or excreta occurs approximately 30 days after infestation and is responsible for the intense pruritus that is the hallmark of the disease.

All household members and close personal contacts should be treated, whether or not they are symptomatic, and patients should be re-examined two weeks after treatment to evaluate effectiveness.

Treatment failures are uncommon. Recurrence of the eruption usually means re-infection has occurred.

Question: 98 of 100

Time taken: 40:12

Theme:Childhood viral infectionsA Adenovirus

B Coxsackie

C Cytomegalovirus

D Epstein Barr virus

E Measles

F Molluscum contagiosum

G Mumps

H Rotavirus

I Rubella

J Varicella

For each patient with the group of symptoms listed below, choose the SINGLE most probable causative agent from the above list of options. Each option may be used once, more than once or not at all.

A three-year old girl presents with a macular confluent rash which appeared initially behind the ears and has spread. Over the previous five days she has had a low grade fever, catarrh and conjunctivitis. Her mother is vague about her immunisation history.

Incorrect - The correct answer is Measles

Measles is caused by RNA paramyxovirus and occurs worldwide. Outbreaks are common in areas with high numbers of non immunized children. Infection is transmitted via respiratory droplets and incubation period is 10-21 days. The prodromal stage fever conjunctivitis, runny nose and coughing lasts for five days. Koplik's spots are bright red lesions with a central white dot which appear on the buccal mucosa. These are virtually diagnostic. The typical macular confluent rash appears on the face from day 3-5 and spreads to the rest of the body. Diagnosis is made from clinical features, viral culture from lesions and a grater than 4-fold rise in antibody titres. Otitis media, pneumonia, meningitis and very rarely several years after primary infection subacute sclerosing panencephalitis (SSPE).

A two-year-old infant boy is admitted to hospital with vomiting, non bloody watery diarrhoea and is dehydrated. It emerges other children from his play group have developed a similar illness.

Incorrect - The correct answer is Rotavirus

Rotavirus is the most common cause of severe viral gastroenteritis worldwide. Infection is via the faeco-oral route and often occurs in children aged between six months to six years. This RNA virus replicates in the intestinal mucosal cells damages transport mechanisms leading to salt and water depletion which results in diarrhorea and vomiting. Diagnosis is made from clinical features and culture of virus from stools and also by polymerase chain reaction techniques. Treatment is mainly re-hydration and correction of any electrolyte imbalance.

A two-year-old boy is mildly unwell. His mother has noticed vesicles in his mouth, palms and soles of his feet.

Incorrect - The correct answer is Coxsackie

Coxsackie A16 virus is the cause of hand, foot and mouth disease characterized by fever, sore throat and ulcerating vesicles in palms, orophaynx and on soles. Incubation period is 5-7 days and these heal without crusting. Treatment is symptomatic.

A ten-year-old girl develops an itchy rash which began on her trunk and has since spread over the entire body. She travelled to see her cousin who was unwell with a painful rash three weeks ago.

Incorrect - The correct answer is Varicella

Varicella (chicken pox) is transmitted by respiratory droplets and contact with somebody with shingles. Incubation period is 14-21 days and following a brief period of malaise, an itchy papulovesicular rash appears on the trunk and spreads to the head and the extremities. The rash evolves from papules to vesicles, pustules and finally crusts. Antiviral therapy is reserved for systemic disease in the immunocompromised.

The four month old baby daughter of an HIV positive mother is admitted to hospital with seizures. She has neonatal jaundice and microcephaly.

Incorrect - The correct answer is Cytomegalovirus

Cytomegalovirus inclusion disease is the result of infection of the foetus. Many organs may be affected and congenital abnormalities result. Microcephaly, seizures, neonatal jaundice, hepatosplenomegaly, deafness and mental retardation are some of the features that may occur.

Question: 99 of 100

Time taken: 40:29

Theme:SplenomegalyA Acute leukaemia

B Autoimmune haemolytic disease

C Cytomegalovirus infection

D Epstein-Barr virus infection

E Neoplasia

F Histiocytosis

G Red cell enzyme defect

H Red cell membrane defect

I Toxoplasmosis

For each scenario choose the most likely diagnosis from the listed options.

A 14-year-old boy presents with a four day history of fever and difficulty in swallowing. On examination he has a scanty erythematous rash, tonsillar pus, tender cervical nodes and 3 cm splenomegaly.

Incorrect - The correct answer is Epstein-Barr virus infection

This boy has a mononucleosis-like illness which is most likely due to EBV.

A 9-month-old boy presents with pallor. On examination he appears well, has a tinge of jaundice and a 3 cm spleen.

Incorrect - The correct answer is Red cell membrane defect

This child has haemolysis, most likely hereditary spherocytosis. In some parts of the world (e.g. the Mediterranean) G6PD deficiency may be more common.

A 3-year-old girl presents with fever, weight loss and irritability. On examination she has a temperature of 38.4°C, neck stiffness, 4 cm hepatomegaly and 5 cm splenomegaly. There are lytic lesions on skull x ray.

Incorrect - The correct answer is Histiocytosis

This girl probably has a malignancy, and the presentation is characteristic of histiocytosis.

In the differential diagnosis of splenomegaly the first thing is to establish whether there is an associated infection. If not, then an FBC and film should be done. This may reveal

underlying haemolysis

atypical lymphocytes

blast cells.

If normal an USS abdomen will be helpful in identifying

congestion

neoplasia

connective tissue

storage disorders.

Question: 100 of 100

Time taken: 43:33

The Köbner's phenomenon is characteristic of:

True / False Pityriasis rosea Incorrect answer selected

Psoriasis Correct

Seborrhoeic dermatitis Incorrect answer selected

Common warts Correct

The rash of systemic juvenile chronic arthritis This is the correct answer

The Köbner phenomenon consists of new dermatological lesions appearing at sites of trauma. It is found in:

Psoriasis.

Epidermyolysis bullosa simplex.

The rash of systemic onset juvenile chronic arthritis.

Common warts.

Lichen planus, lichen niditus, lichen sclerosis.

Copyright © 2002 Dr Colin Melville

Question: 1 of 11

Time taken: 00:15

Is/are the following true of scabies?

True / False It can be spread by droplet infection. Incorrect answer selected

It causes itchiness in the skin even where there is no obvious lesion to be seen. Correct

It is best treated by salicylate emulsion. Incorrect answer selected

It is caused by Staph. aureus. Incorrect answer selected

Typically affects the face. Incorrect answer selected

Scabies typically affects the interdigital webs and skin folds of the arms and legs.

The itch may persist for two to three weeks after successful treatment.

It is caused by the mite Sarcoptes scabiei.

Permethrin-containing lotions are the treatment.

It is spread by skin contact and sharing clothes and bedding.

Question: 2 of 11

Time taken: 00:33

Theme:Vaginal discharge.A Allergic

B Bacterial vaginosis

C Child sex abuse

D Foreign body

E Pinworms

F Rectovaginal fistula

G Tumour

H Vulvovaginitis, infectious

I Vulvovaginitis, non-specific

For each scenario choose the most likely diagnosis:

A 4-year-old girl presents with persistent scratching of her anus. Perineal examination is unremarkable.

Incorrect - The correct answer is Pinworms

Perianal itching is usually due to pinworms (threadworms). This can be confirmed by a sellotape test, where a strip of sellotape is placed at the anal margin on waking and examined for worms.

A 4-year-old girl presents with vaginal irritation and scanty discharge. On examination she has minimal perineal redness.

Incorrect - The correct answer is Vulvovaginitis, non-specific

Vaginal irritation with redness is common as girls learn to wipe themselves after defecation and as the skin is thin and sensitive at this age. Avoidance of occlusion (for example, plastic pants), irritants (for example, bubble bath) and keeping the perineum dry usually results in rapid resolution.

A 9-year-old girl presents with vaginal irritation and offensive discharge. On examination a mucopurulent discharge is seen.

Incorrect - The correct answer is Vulvovaginitis, infectious

Offensive mucopurulent discharge suggests a significant infection, such as Chlamydia or gonorrhoea. Swabs should be taken, including for these organisms. If positive then child sex abuse has been confirmed.

Question: 3 of 11

Time taken: 00:40

A 7-year-old girl presents with a florid rash over her face and trunk. She has had a sore throat treated for seven days by her family doctor with amoxicillin. Otherwise she has been well.

She was born at term weighing 3.26 kg and there were no neonatal problems. She is fully immunised and there is no family history or social history of note.

On examination she is apyrexial and well. She has a florid confluent blanching pink rash all over her face and trunk, with less on her limbs. ENT examination is unremarkable.

What is the most likely diagnosis?

(Please select 1 option) Drug reaction Correct

Erythema multiforme

Infectious mononucleosis

Kawasaki disease

Scarlet fever

The history of florid rash seven days after amoxicillin suggests a drug reaction.

If Epstein-Barr virus (EBV) is present a rash is seen in 90%, and appears within a day or two of amoxicillin starting.

The rash will disappear even if amoxicillin is continued, suggesting that it does not have an allergic basis.

Question: 4 of 11

Time taken: 00:48

An 8-month-old child presents with spots on the legs.

He is well and feeding well. 39+6/40 3.5 kg, no neonatal problems. No drugs nor medications, fully immunised. No FH/SH of note.

On examination temperature 37.4°C (tympanic), RR 30/min, HR 110/min. Well perfused, capillary refill time of 1 second. 20-30 1-2 mm non-blanching purpuric spots over the shins.

What is the most likely diagnosis?

(Please select 1 option) Child physical abuse Incorrect answer selected

Cough petechiae

Enteroviral infection This is the correct answer

Henoch-Schoenlein purpura

Idiopathic thrombocytopenic purpura

This child is well, and presents with purpuric spots and a low-grade fever.

Although about 20% of such children have serious bacterial infection and 7-10% have meningitis/ septicaemia, this still leaves 70% who have some sort of viral infection.

A large number of viruses (for example, varicella and Epstein-Barr virus [EBV]) can present in this way, although in clinical practice the specific cause is rarely found.

Question: 5 of 11

Time taken: 01:13

Theme:Mucocutaneous infections and infestationsA Enterobius vermicularis

B Chlamydia

C Candidiasis

D Tinea pedis

E Pityriasis rosea

F Scabies

G Ringworm

H Lichen sclerosis

I Napkin dermatitis

J Lichen planus

For each of the following children who present with an itch, select the most appropriate diagnosis from the list of options:

A child complains of an itchy rash over his arms. His brother and sister have similar symptoms.

Incorrect - The correct answer is Scabies

Scabies is caused by the mite Sarcoptes scabiei hominis. Transmission is through close body contact, the adult mites lay their eggs in burrows in the skin and it results in an eczematous rash with parotitis skin.

A baby has an excoriated perineal rash involving the flexures with satellite lesions.

Incorrect - The correct answer is Candidiasis

Candidiasis is caused by Candida albicans (yeast). In babies it presents as a perineal rash which usually affects the flexures. Satellite lesions may also be seen.

A boy has an itchy rash over the trunk with a solitary large oval lesion on the back.

Incorrect - The correct answer is Pityriasis rosea

Pityriasis rosea is a benign condition of the skin resulting in oval pink/brown scaly lesions over the trunk, which are usually preceded by a herald patch (a solitary large lesion usually between 1-10 cm). No treatment is required.

Question: 6 of 11

Time taken: 01:30

Theme:Skin lesionsA Café au lait spots

B Capillary haemangioma

C Port-wine stain

D Shagreen patch

E Adenoma sebaceum

F Cutis marmorata

G Herald patch

H Perioral pigmentation

I Hypopigmentation

J Tuber

Select one option from the list that is most suitable for the following patients:

A child has an abundance of birth marks associated with axillary freckling.

Correct

This relates to neurofibromatosis type 1. Café au lait lesions are brown pigmented marks found in abundance in neurofibromatosis as well as other genetic conditions, for example, tuberous sclerosis.

A child with learning difficulties is described as having goose-like flesh.

Incorrect - The correct answer is Shagreen patch

This relates to tuberous sclerosis. It is a condition whereby hamartomatous lesions become evident in early childhood. Seizures and learning difficulties are common and skin manifestations include fibrous angiomatous lesions in the nasolabial folds known as adenoma sebaceum. White macules are seen over the trunk known as ash leaf macules and shagreen patches resemble goose-like flesh. Subungual fibromata are also common.

A child with a history of iron deficiency anaemia presents with per rectal bleeding and is found to have this characteristic appearance.

Incorrect - The correct answer is Perioral pigmentation

This refers to Peutz-Jegher's syndrome characterised by perioral pigmentation. There is a history of iron deficiency anaemia and colonoscopy may reveal polyps.

Question: 7 of 11

Time taken: 21:37

A 7-year-old girl presents with a florid rash over her face and trunk.

She has had a sore throat treated for seven days by her family doctor with amoxicillin. Otherwise she has been well. She was born at term weighing 3.26 kg and there were no neonatal problems. She is fully immunised and there is no FH/SH of note.

On examination she is apyrexial and well. She has a florid confluent blanching pink rash all over her face and trunk, with less on her limbs. ENT examination is unremarkable.

What is the most likely diagnosis?

(Please select 1 option) Contact dermatitis Incorrect answer selected

Drug reaction This is the correct answer

Erythema multiforme

Impetigo

Viral exanthems

The history of florid rash seven days after amoxicillin suggests a drug reaction.

If Epstein-Barr virus (EBV) is present a rash is seen in 90%, and appears within a day or two of amoxicillin starting.

The rash will disappear even if amoxicillin is continued, suggesting that it does not have an allergic basis.

Question: 8 of 11

Time taken: 21:50

Are the following characteristic features of dermatitis herpetiformis?

True / False Association with HLA-B27 in 25% of patients. Incorrect answer selected

IgA and C3 detected in the dermoepidermal junction by immunoflouresence studies. Correct

Onset in infancy. Incorrect answer selected

Painless skin lesions. Incorrect answer selected

Subepidermal blisters with a predominant neutrophil infiltrate. Correct

Onset is usually between two and seven years, and the lesions are painful but pleomorphic, including urticarial, papular, vesicular or bullous lesions.

These affect particularly the knees, elbows, shoulders, buttocks and scalp. The mucus membranes are usually spared.

There may be visible excoriation.

Gluten-sensitive enteropathy is found in 85%, and the lesions show neutrophil infiltrate with IgA and C3 deposition in the dermoepidermal junction on IF.

There is a strong association with HLA-B8.

Oral dapsone provides immediate relief form the intense pruritis.

The enteropathy responds more rapidly to a gluten-free diet than the skin lesions.

Copyright © 2002 Dr Colin Melville

Question: 9 of 11

Time taken: 22:03

Regarding tinea capitis, which of the following statements is/are true?

True / False It causes patches that fluoresce dull green under Wood's lamp. Incorrect answer selected

It is effectively treated with topical nystatin ointment. Incorrect answer selected

It is most commonly caused by the fungus microsporum canis. Incorrect answer selected

It often results in permanent alopecia. Incorrect answer selected

Its presence should suggest immunological deficiency. Incorrect answer selected

Tinea capitus is a dermatophyte infection of the scalp most often caused by Trichophyton tonsurans, and occasionally by Microsporum canis.

It is commonest in areas of socio-economic depravation.

M. canis is a zoophilic species acquired from cats and dogs. There is initially a small papule at the base of the hair follicle which spread peripherally forming a scaly circular plaque (ringworm) within which there are brittle, broken infected hairs (exclamation mark hairs).

Confluent patches of alopecia develop and there may be pruritis.

Sometimes a severe inflammatory response produces an elevated boggy granulomatous mass (kerion), studded with sterile pustules.

There may be fever and regional lymphadenopathy, and occasionally permanent scarring and alopecia may result.

The crusted patches of T. tonsurans do not fluoresce under Wood's light.

Microscopic examination of a potassium hydroxide (KOH) preparation shows tiny spores and the fungi may be grown in Sabouraud medium with antibiotics.

Oral griseofulvin for two to three months is required, or ketoconazole for resistant cases.

Copyright © 2002 Dr Colin Melville

Question: 10 of 11

Time taken: 22:19

Molluscum contagiosum:

True / False is a DNA virus of the pox family Correct

is found only in humans Correct

produces severely pruritic lesions Incorrect answer selected

produces characteristic paracentric umbilication of the papules Correct

responds favourably to cantharadin application Correct

Molluscum contagiosum is a superficial skin infection. The virus invades the skin causing the appearance of firm, flesh-coloured, doughnut-shaped bumps, about 2-5 mm in diameter which are usually non-pruritic. Molluscum contagiosum is caused by a virus belonging to the poxvirus family. Close physical contact is usually necessary for transmission; indirect transmission from shared towels, swimming pools, etc., may also be responsible for infection. The incubation period varies from several weeks to several months. Treatment is with surgical excision, cryotherapy, trichloroacetic acid or cantharidin topical therapies.

Question: 11 of 11

Time taken: 22:27

A 16-year-old girl is seen in clinic as she is concerned due to areas of hair loss on the scalp. Past medical history includes atopic eczema and she has a number of depigmented areas on her hands. What is the most likely diagnosis?

(Please select 1 option) Alopecia areata Correct

Hypothyroidism

Seborrhoeic dermatitis

Systemic lupus erythematosus

Trichotillomania

This girl has a combination of vitiligo and alopecia areata which can co-exist and have similar autoimmune aetiology. Discrete areas of hair loss and normal texture on the scalp are highly suggestive of alopecia areata.